База 2017 англ

Download as pdf or txt
Download as pdf or txt
You are on page 1of 47

Завантажено з сайту https://тестування.

укр/ - онлайн тестування КРОК

ТЕСТУВАННЯ.УКР Бази тестів

Буклет 2017 року

Цей тест можна пройти в режимі онлайн тестування на сайті https://тестування.укр/testkrok/studing/758

Це офіційні тести з сайту Центру тестування https://www.testcentr.org.ua/

1. A 32-year-old welder complains of weakness and fever. His illness initially presented as
tonsillitis one month earlier. On examination: body temperature - 38,9oC, RR-24/min., HR-
100/min., BP- 100/70 mm Hg, hemorrhages on the legs, enlargement of the lymph nodes.
Complete blood count: Hb- 70 g/l, RBC- 2,2 • 1012/l, WBC- 3,0 • 108 9/l with 32% of blasts, 1%
of eosinophils, 3% of band neutrophils, 36% of segments, 20% of lymphocytes, and 8% of
monocytes, ESR- 47 mm/hour. What is the cause of anemia?
A. Acute leukemia*
B. Chronic lympholeukemia
C. Aplastic anema
D. B12-deficient anemia
E. Chronic hemolytic anemia

2. After a 5-day-long celebration of his daughter’s wedding a 65-year-old patient ”saw” in his
yard many cats, chickens, and rats. He tried to chase them away, but was scared off when the
animals started to scold him and tried to harm him. Make the diagnosis:
A. Delirium tremens*
B. Senile psychosis
C. Schizophrenia
D. Organic brain syndrome
E. Reactive hallucinosis

3. An 8-year-old boy developed a temperature of 37, 5oC two days after his recovery from the
case of URTI. He complains of suffocation, heart pain. Objectively: the skin is pale,
tachycardia, the I heart sound is weakened, short systolyc murmur in the 4th intercostal area
near the left edge of the breastbone. What heart disorder such clincal presentation is
characteristic of?
A. Nonrheumatic myocarditis*
B. Primary rheumatic carditis
C. Myocardiodystrophy
D. Fallot’s tetrad
E. Cardiomyopathy

4. A 28-year-old woman has been delivered to a hospital with acute pain in the lower abdomen.
There was a brief syncope. The delay of menstruation is 2 months. Objectively: the patient has
pale skin, BP- 90/50 mm Hg, Ps- 110/min. Lower abdomen is extremely painful. Vaginal
examination reveals uterus enlargement. Promtov’s sign (pain during bimanual gynecological
examination) is positive. Right uterine appendages are enlarged and very painful. Posterior
vault hangs over. What is the most likely diagnosis?
A. Right-sided tubal pregnancy*
B. Right ovary apoplexy
C. Acute right-sided salpingoophoritis

1
Завантажено з сайту https://тестування.укр/ - онлайн тестування КРОК

D. Pelvioperitonitis
E. Incipient abortion

5. 5 weeks after hypothermia a 22-year-old patient developed fever, weakness, muscle pain,
inability to move independently. Objectively: tenderness, induration of shoulder and shin
muscles, restricted active movements, erythema on the anterior surface of the chest. There is
a periorbital edema with heliotropic erythema. Gottron’s sign is present. What investigation is
required for the diagnosis verification?
A. Muscle biopsy*
B. Aminotransferase activity
C. Pneumoarthrography
D. ASO titer
E. Rheumatoid factor

6. A 37-year-old woman complains of headaches, nausea, vomiting, spasms. The onset of the
disease occurred the day before due to her overexposure to cold. Objectively: fever up to
40oC; somnolence; rigid neck; Kernig’s symptom is positive on the both sides; general
hyperesthesia. Blood test: leucocytosis, increased ESR. Cerebrospinal fluid is turbid, yellow-
tinted. What changes of the cerebrospinal fluid are most likely?
A. Neutrophilic pleocytosis*
B. Lymphocytic pleocytosis
C. Blood in the cerebrospinal fluid
D. Xanthochromia in the cerebrospinal fluid
E. Albuminocytological dissociation

7. A 44-year-old patient complains of difficult urination, sensation of incomplete urinary bladder


emptying. Sonographic examination of the urinary bladder near the urethra entrance revealed
an oval well-defined hyperechogenic formation 2x3 cm large that was changing its position
during the examination. What conclusion can be made?
A. Stone*
B. Malignant tumour of the urinary bladder
C. Urinary bladder polyp
D. Prostate adenoma
E. Primary ureter tumour

8. 4 days after a patient received a gunshot wound to the soft tissues of middle third of the thigh,
his condition suddenly began deteriorating. There are complaints of bursting pain in the
wound; pain has been increasing during the last 12 hours. Edema of skin and hypodermic
tissue quickly grows. Body temperature is 38,2oC, heart rate is 102/min. The wound edges
gape, are dull in color; the muscles, viable as of day before, now protrude into the wound, look
boiled, are dull in color, have dirty-gray coating, and fall apart when held with forceps. What
infection has developed in the wound?
A. Anaerobic*
B. Aerobic gram-negative
C. Putrid
D. Aerobic gram-positive
E. Diphtheria of the wound

2
Завантажено з сайту https://тестування.укр/ - онлайн тестування КРОК

9. A 35-year-old patient has been suffering from an illness for 3 days. 5 days ago he returned
from a trip to Africa. The onset of disease was accompanied by fever up to 40oC, chills, acute
headache, myalgia. In the axillary region the lymph node enlarged up to 3x6 cm can be
palpated. The lymph node is dense, intensely painful, slightly mobile, without clear margins;
the skin over the node is hyperenic and tight. Tachycardia is present. Make the preliminary
diagnosis:
A. Plague*
B. Sepsis
C. Tularemia
D. Lymphadenitis
E. Anthrax

10. A parturient woman complains of pain in her mammary gland. In the painful area there is an
infiltration 3x4 cm in size with softened center. Body temperature is 38,5oC. What is the most
likely diagnosis?
A. Acute suppurative mastitis*
B. Pneumonia
C. Pleurisy
D. Milk retention
E. Birth trauma

11. A 52-year-old patient, who has been suffering from angina pectoris, for 2 weeks has
increasingly frequent pain attacks in the area behind his sternum and his need for
nitroglycerine has increased. Objectively: the condition is of moderate severity. The skin is
pale. Heart sounds are weakened, rhythmic. Heart rate is 84/min. ECG shows no signs of focal
myocardial injury. What is the most likely diagnosis?
A. Progressive angina pectoris*
B. First-time angina pectoris
C. Stable NYHA functional class II angina pectoris
D. Variant angina pectoris
E. Acute cardiac infarction

12. A 9-year-old boy has been suffering from multiple bronchiectasis since he was 3 years old.
Exacerbations occur frequently (34 times a year), after conservative therapy there are short
remission periods. The disease progresses, the child is physically underdeveloped, presents
with pale skin, acrocyanosis, deformed nail plates in the shape of ”clock-face” Bronchography
reveals saccular bronchiectases in the lower lobe of the right lung. What further treatment
tactics should be chosen?
A. Surgical intervention*
B. Continuation of conservative therapy
C. Physiotherapy
D. Sanatorium-and-spa treatment
E. Physical training

13. Caries morbidity rate is 89% among residents of a community. It is determined that fluorine
content in water is 0,1 mg/l. What preventive measures should be taken?
A. Water fluorination*

3
Завантажено з сайту https://тестування.укр/ - онлайн тестування КРОК

B. Tooth brushing
C. Fluorine inhalations
D. Sealant application
E. Introduction of more vegetables to the diet

14. A patient received flame burns of both hands. On the dorsal and palmar surface of the hands
there are blisters filled with serous fluid. The wrist joint region is hyperemic. The forearms
were not injured. What is the provisional diagnosis?
A. II degree flame burn of the hands, 4% of body surface area*
B. II degree flame burn of the hands, 2% of body surface area
C. IIIa degree flame burn of the hands, 4% of body surface area
D. III degree flame burn of the hands, 4% of body surface area
E. IIb degree flame burn of the hands, 2% of body surface area

15. During the periodic medical examination an assembly fitter (works on soldering details) didn’t
report any health problems. Closer examination revealed signs of asthenic-vegetative
syndrome. Blood included red blood cells with basophilic aggregations and a somewhat higher
number of reticulocytes, urine had a high concentration of delta-aminolevulinic acid. The
complex of symptoms indicates the initial stage of chronic intoxication with:
A. Lead*
B. Manganese
C. Mercury
D. Tin
E. Ethanol

16. A 29-year-old woman came to a gynecologist with complaints of irritability, tearfulness,


headache, nausea, occasional vomiting, pain in the heart area, tachycardia attacks, memory
impairment, meteorism. These signs appear 6 days before menstruation and disappear the day
before menstruation or during its first 2 days. On vaginal examination: the uterus and uterine
appendages are without alterations. What diagnosis is the most likely?
A. Premenstrual syndrome*
B. Algodismenorrhea
C. Ovarian apoplexy
D. Genital endometriosis
E. Neurosis

17. Examination of a group of persons living on the same territory revealed the following common
symptoms: dark-yellow pigmentation of the tooth enamel, diffuse osteoporosis of bone
apparatus, ossification of ligaments and joints, functional disorders of the central nervous
system. This condition may be caused by the excessive concentration of the following
microelement in food or drinking water:
A. Fluorine*
B. Copper
C. Nickel
D. Iodine
E. Cesium

4
Завантажено з сайту https://тестування.укр/ - онлайн тестування КРОК

18. An infant has been born at the 41st week of gestation. The pregnancy was complicated with
severe gestosis of the second semester. The weight of the baby is 2400 g, the height is 50 cm.
Objectively: the skin is flabby, the layer of subcutaneous fat is thin, hypomyotonia is observed,
neonatal reflexes are weak. The internal organs are without pathologic alterations. This
newborn can be assessed as a:
A. Full-term infant with prenatal growth retardation*
B. Premature infant
C. Immature infant
D. Postmature infant
E. Full-term infant with normal body weight

19. A patient suffering from infiltrative pulmonary tuberculosis was prescribed streptomycin,
rifampicin, isoniazid, pyrazi-namide, vitamin C. One month after the beginning of the
treatment the patient started complaining of reduced hearing and tinnitus. What drug has
such a side effect?
A. Streptomycin*
B. Isoniazid
C. Rifampicin
D. Pyrazinamide
E. Vitamin C

20. A 39-year-old man complains of morning headaches, appetite loss, nausea, morning vomiting,
periodic nasal hemorrhages. The patient had a case of acute glomerulonephritis at the age of
15. Examination revealed rise of arterial pressure up to 220/130 mm Hg, skin hemorrhages on
his arms and legs, pallor of skin and mucous membranes. What biochemical parameter is the
most important for making diagnosis in this case?
A. Blood creatinine*
B. Blood bilirubin
C. Blood sodium
D. Uric acid
E. Fibrinogen

21. A worker of a glass-blowing workshop complains of headache, irritability, visual impairment -


he sees everything as if through a ”net” Objectively: hyperemic sclera, thickened cornea,
decreased opacity of pupils, visual acuity is 0,8 in the left eye, 0,7 in the right eye. The worker
uses no means of personal protection. What is the most likely diagnosis?
A. Cataract*
B. Conjunctivitis
C. Keratitis
D. Blepharospasm
E. Progressive myopia

22. For a week a 42-year-old patient has been suffering from fever attacks followed by high
temperature, which occur each 48 hours. Body temperature raises up to 40°C and decreases in
3-4 hours with excessive sweating. The patient presents with loss of appetite and general
fatigue. The skin is pale and sallow. The liver and spleen are enlarged and dense on palpation.
What method of diagnosis verification would be most efficient?

5
Завантажено з сайту https://тестування.укр/ - онлайн тестування КРОК

A. Microscopy of blood smear and thick blood film*


B. Complete blood count
C. Bacteriological analysis
D. Immune-enzyme assay
E. Microscopy of hanging blood drop

23. A 28-year-old patient is a drug addict. He has been sick for a year, when noticed general
weakness, increased sweating, and weight loss. He often had cases of respiratory diseases.
Within the last 2 days he demonstrates intermittent fever with profuse night sweating,
increased general weakness, developed diarrhea with mucus and blood admixtures. On
examination: polylymphadenopathy, herpetic rashes in the oral cavity; on abdominal palpation:
the liver and spleen are enlarged. What is the most likely diagnosis?
A. HIV-infection*
B. Herpetic stomatitis
C. Chronic lymphatic leukemia
D. Colon cancer
E. Chronic sepsis

24. A 40-year-old patient has acute onset of disease caused by overexposure to cold. Temperature
has increased up to 39oC. Foul-smelling sputum is expectorated during coughig. Various moist
crackles can be auscultated above the 3rd segment on the right. Blood test: leukocytes - 15,0 •
109/l, stab neutrophils - 12%, ESR- 52 mm/hour. On X-ray: in the 3rd segment on the right
there is a focus of shadow 3 cm in diameter, low density, with fuzzy smooth margins and a
clearing in its center. What disease is most likely in the given case?
A. Pneumonia complicated by an abscess*
B. Infiltrative tuberculosis
C. Peripheral pulmonary cancer
D. Cystic echinococcosis
E. Pulmonary cyst

25. A 48-year-old patient was found to have diffuse enlargement of the thyroid gland,
exophthalmia, weight loss of 4 kg in 2 months, sweating. Objectively: HR- 105/min, BP-140/70
mm Hg. Defecation act is normal. What kind of therapy is recommended in this case?
A. Mercazolil*
B. Radioactive iodine
C. Propranolol
D. Lugol’s solution
E. Thyroxine

26. A 26-year-old woman is suspected to suffer from systemic lupus erythematosus due to systemic
lesions of skin, vessels, joints, serous tunics, and heart that developed after photosensitization.
The following is detected: LE cells, antibodies to native DNA, isolated anti-centromere
antibodies, rheumatoid factor is 1:100, Wassermann reaction is positive, circulating immune
complex is 120 units. What immunological indicators are considered to be specific to this
disease?
A. DNA antibodies*
B. Rheumatoid factor

6
Завантажено з сайту https://тестування.укр/ - онлайн тестування КРОК

C. Anti-centromere antibodies
D. Immunoglobulin A
E. Increased circulating immune complex

27. A woman came to a doctor with complaints of increased body temperature up to 37,8oC and
moderately sore throat for the last 3 days. Objectively: mandibular lymph nodes are enlarged
up to 3 cm. Palatine tonsils are hypertrophied, with gray coating that spreads to the uvula and
anterior pillars of the fauces. What is the most likely diagnosis?
A. Oropharyngeal diphtheria*
B. Infectious mononucleosis
C. Pseudomembranous (Vincent’s) tonsillitis
D. Agranulocytosis
E. Oropharyngeal candidiasis

28. A primigravida at the term of 20 weeks complains of pain in her lower abdomen, smearing
blood-streaked discharge from the genital tracts. Uterine tone is increased, fetus is mobile. On
vaginal examination: the uterus is enlarged according to the term, uterine cervix is shortened
to 0,5 cm, external cervical orifice is open by 2 cm. What is the most likely diagnosis?
A. Risk of late abortion with hemorrhage*
B. Risk of late abortion without hemorrhage
C. The process of late abortion
D. Incomplete late abortion
E. Attempted late abortion

29. A 65-year-old man was diagnosed with B12-deficient anemia and the treatment was
prescribed. A week later control blood test was performed. What would be the early indicator
of the therapy effectiveness?
A. Increased number of reticulocytes*
B. Increased hemoglobin level
C. Megaloblastic hematopoiesis
D. Normoblastic hematopoiesis
E. Increased erythrocyte number

30. A 35-year-old woman complains of heart pain (”aching and drilling”) occurring mainly in the
morning in autumn and spring and irradiating to the neck, back and abdomen; rapid
heartbeat; low vitality. Occurrence of this condition is not associated with physical activity. In
the evening, the patient’s condition improves. Study of somatic and neurological status and
ECG reveal no pathology. What pathology is most likely to have caused these clinical
presentations?
A. Somatization depression*
B. Resting stenocardia
C. Pseudoneurotic schizophrenia
D. Neurocirculatory asthenia
E. Hypochondriacal depression

31. On the 4th day after recovery from a cold a patient was hospitalized with complaints of solitary

7
Завантажено з сайту https://тестування.укр/ - онлайн тестування КРОК

spittings of mucoid sputum. On the 2nd day there was a single discharge of about 250 ml of
purulent blood-streaked sputum. Objectively: the patient’s condition is moderately severe.
Respiratory rate -28-30/min., Ps- 96/min., BP- 110/70 mm Hg. Respiration over the left lung is
vesicular, over the right lung - weakened. There are various moist crackles over the lower lobe
and amphoric breath sounds near the angle of scapula. What is the most likely diagnosis?
A. Acute pulmonary abscess*
B. Exudative pleuritis
C. Acute focal pneumonia
D. Pleural empyema
E. Pyopneumothorax

32. A 65-year-old woman on abdominal palpation presents with a tumor in the umbilical region
and above it; the tumor is 13x8 cm in size, moderately painful, non-mobile, pulsing. On
auscultation systolic murmur can be observed. What is the most likely diagnosis?
A. Abdominal aneurysm*
B. Gastric tumor
C. Arteriovenous aneurysm
D. Tricuspid insufficiency
E. Bicuspid insufficiency

33. A 45-year-old man complains of cough fits and tickling in his nasopharynx. He had been
staying for 10 days in the polluted area created by the Chornobyl nuclear power plant
accident. Rhinoscopy shows signs of severe nasopharynx irritation. What radionuclide is the
cause of this irritation?
A. Radioactive iodine*
B. Radioactive cesium
C. Radioactive strontium
D. Radioactive plutonium
E. Radioactive cobalt

34. A 20-year-old patient complains of severe headache, double vision, weakness, fever, irritability.
Objectively: body temperature is 38,1oC, the patient is reluctant to contact, sensitive to
stimuli. There are ptosis of the left eyelid, exotropia, anisocoria S>D, pronounced meningeal
syndrome. On lumbar puncture the cerebrospinal fluid flowed out under a pressure of 300 mm
Hg, the fluid is clear, slightly opalescent. 24 hours later there appeared fibrin film. Protein -
1,4 g/l, lymphocytes - 600/3 per mm3, sugar - 0,3 mmol/l. What is the provisional diagnosis?
A. Tuberculous meningitis*
B. Meningococcal meningitis
C. Lymphocytic (Armstrong’s) meningitis
D. Syphilitic meningitis
E. Mumps meningitis

35. A 37-year-old worker during a fire ended up in the area of high CO concentration. He was
delivered to a hospital in unconscious state. Objectively: the skin of his face and hands is
crimson. Respiration rate is 20/min. ECG: alterations specific for hypoxic myocardium. Hourly
diuresis is 40 ml. Blood test: erythrocytes - 4,5 • 1012/l, Hb- 136 g/l, color index - 0,89, ESR- 3
mm/hour, carboxyhemoglobin - 5%. What criterion allows determining the severity of the

8
Завантажено з сайту https://тестування.укр/ - онлайн тестування КРОК

patient’s condition?
A. Carboxyhemoglobin concentration*
B. Respiratory disorders
C. ECG results
D. Extent of trophic disorders
E. Development of chronic renal failure

36. After a case of purulent otitis a 1-year-old boy has developed pain in the upper third of the left
thigh, body temperature up to 39oC. Objectively: swelling of the thigh in its upper third and
smoothed out inguinal fold. The limb is in semiflexed position. Active and passive movements
are impossible due to severe pain. What diagnosis is the most likely?
A. Acute hematogenous osteomyelitis*
B. Acute coxitis
C. Intermuscular phlegmon
D. Osteosarcoma
E. Brodie’s abscess

37. A patient with suspected pheochromocytoma has normal blood pressure in the periods
between the atacks and a tendency towards tachycardia. Urine test revealed no pathologies. It
was decided to use a provocative test with histamine. What medication should be prepared to
provide emergency care in case of positive test result?
A. Phentolamine*
B. Pipolphen
C. Nifedipine
D. Mesatonum
E. Prednisolone

38. A 42-year-old woman complains of severe pulsing headache in the frontoparietal area, vertigo,
palpitations. She has been suffering from hypertension for 3 years. Significant increase of BP
occurs 2-3 times per month and lasts for 3-8 hours. The left ventricle is enlarged, heart sounds
are clear, heart rate - 105/min., BP- 225/115 mm Hg. ECG: signs of left ventricular
hypertrophy. What drug would be the most effective for termination of cerebral crisis attack?
A. Labetalol*
B. Hydrochlorothiazide (Hypothiazide)
C. Captopril
D. Losartan
E. Clonidine (Clophelin)

39. A 45-year-old patient complains of pain in the epigastric region, left subcostal area, abdominal
distension, diarrhea, loss of weight. He has been suffering from this condition for 5 years.
Objectively: the tongue is moist with white coating near the root; deep palpation of abdomen
reveals slight pain in the epigastric region and Mayo-Robson’s point. Liver is painless and
protrudes by 1 cm from the costal arch. Spleen cannot be palpated. What disease can be
primarily suspected?
A. Chronic pancreatitis*
B. Atrophic gastritis
C. Peptic stomach ulcer

9
Завантажено з сайту https://тестування.укр/ - онлайн тестування КРОК

D. Chronic cholecystitis
E. Chronic enteritis

40. For three years a 31-year-old woman has been complaining of pain and swelling of her
radiocarpal and metacarpophalangeal articulations, their reduced mobility in the morning,
which persisted up to 1,5 hours. Two weeks ago she developed pain, swelling, and reddening
of her knee joints, her body temperature increased up to 37,5oC. The treatment was belated.
Examination of the internal organs revealed no pathologic alterations. Diagnosis of rheumatoid
arthritis was made. What alterations are the most likely to be visible on the arthrogram?
A. Joint space narrowing, usuration*
B. Joint space narrowing, subchondral osteosclerosis
C. Cysts in the subchondral bone
D. Numerous marginal osteophytes
E. Epiphyseal osteolysis

41. A 58-year-old woman complains of spontaneous bruises, weakness, bleeding gums, dizziness.
Objectively: the mucous membranes and skin are pale with numerous hemorrhages of various
time of origin. Lymph nodes are not enlarged. Ps- 100/min, BP-110/70 mm Hg. There are no
alterations of internal organs. Blood test results: RBC-3,0 • 1012/l, Hb- 92 g/l, color index - 0,9,
anisocytosis, poikilocytosis, WBC- 10 • 109/l, eosinophils - 2%, stab neutrophils - 12%,
segmented neutrophils - 68%, lymphocytes -11%, monocytes - 7%, ESR-12 mm/h. What
laboratory test should be performed additionally to make the diagnosis?
A. Platelets*
B. Reticulocytes
C. Clotting time
D. Osmotic resistance of erythrocytes
E. Fibrinogen

42. After tonsillectomy a woman with systemic lupus erythematosus, who has been taking
prednisolone for a year, developed acute weakness, nausea, vomiting, pain in the right iliac
area, watery stool up to 5 times per day. Ps- 96/min., BP- 80/50 mm Hg. What preventive
therapy should have been administered prior to the surgery?
A. Increase of prednisolone dosage*
B. Prescription of desoxycorticosterone acetate
C. Prescription of antibiotics
D. Administration of Ringer’s solution
E. Administration of 10% NaCl solution

43. A 45-year-old woman complains of paroxysmal intolerable facial pain on the left with attacks
that last for 1-2 minutes. Attacks are provoked by chewing. The disease onset was two month
ago after overexposure to cold. Objectively: pain at the exit points of the trigeminal nerve on
the left. Touching near the wing of nose on the left induces new pain attack with tonic spasm
of the facial muscles. What is the most likely diagnosis among those listed?
A. Trigeminal neuralgia*
B. Glossopharyngeal neuralgia
C. Temporomandibular joint arthritis
D. Facial migraine

10
Завантажено з сайту https://тестування.укр/ - онлайн тестування КРОК

E. Maxillary sinusitis

44. A 28-year-old man complains of skin rash and itching on the both of his hands. The condition
persists for 1,5 years. The exacerbation of his condition he ascribes to the occupational
contact with formaldehyde resins. Objectively: lesion foci are symmetrically localized on both
hands. Against the background of erythema with blurred margins there are papulae, vesicles,
erosions, crusts, and scales. What is the most likely pathology?
A. Occupational eczema*
B. Idiopathic eczema
C. Allergic dermatitis
D. Simple contact dermatitis
E. Erythema multiforme

45. A 10-year-old boy with symptoms of arthritis and myocarditis was delivered into a hospital.
Based on clinical examination the preliminary diagnosis of juvenile rheumatoid arthritis was
made. What symptom is the most contributive for the diagnostics of this disease?
A. Reduced mobility of the joints in the morning*
B. Regional hyperemia of the joints
C. Affection of the large joints
D. Enlarged heart
E. Increased heart rate

46. A 25-year-old patient was delivered to an infectious diseases unit on the 3rd day of illness with
complaints of headache, pain in the lumbar spine and gastrocnemius muscles, high fever,
chills. Objectively: condition is of moderate severity. Scleras are icteric. Pharynx is hyperemic.
Tongue is dry with dry brown coating. Abdomen is distended. Liver is enlarged by 2 cm.
Spleen is not enlarged. Palpation of muscles, especially gastrocnemius muscles, is painful.
Urine is dark in color. Feces are normal in color. What is the most likely diagnosis?
A. Leptospirosis*
B. Viral hepatitis type A
C. Malaria
D. Infectious mononucleosis
E. Yersiniosis

47. A 28-year-old woman complains of skin hemorrhages after minor traumas and spontaneous
appearance of hemorrhages on the front of her torso and extremities. On examination: the skin
is variegated (old and new hemorrhages), bleeding gums. Blood platelets - 20 109/l; in the
bone marrow there is increased number of megakaryocytes and no platelet production.
Treatment with steroid hormones was effective. What disease is it?
A. Idiopathic thrombocytopenic purpura*
B. Hemophilia
C. Rendu-Osler-Weber disease (Hereditary hemorrhagic telangiectasia)
D. Disseminated intravascular coagulation
E. Acute vascular purpura

48. A 74-year-old patient was delivered into admission room with clinical presentations of acute

11
Завантажено з сайту https://тестування.укр/ - онлайн тестування КРОК

deep vein thrombosis of the shin. What symptom is the most typical of this pathology?
A. Homans’ sign*
B. Rovsing’s sign
C. Courvoisier’s sign
D. Mayo-Robson’s sign
E. Grey Turner’s sign

49. A 9-month-old child presents with fever, cough, dyspnea. The symptoms appeared 5 days ago
after a contact with a person with URTl. Objectively: the child is in grave condition.
Temperature is 38oC, cyanosis of nasolabial triangle is present. rR- 54/min, nasal flaring
during breathing is observed. There was percussion dullness on the right below the scapula
angle and tympanic sound over the other areas of lungs. Auscultation revealed bilateral fine
moist crackles predominating on the right. What is the most likely diagnosis?
A. Pneumonia*
B. URTI
C. Acute laryngotracheitis
D. Acute bronchitis
E. Acute bronchiolitis

50. The mother of a 3-month-old child came to a family doctor with complaints of her child being
physically underdeveloped and suffering from cough attacks and dyspnea. Anamnesis: the
child is the result of the second full-term pregnancy with the risk of miscarriage (the first child
died of pulmonary pathology at the age of 4 months, according to the mother). Body mass at
birth is 2500 g. Cough attacks were observed from the first days of life, twice the child was
treated for bronchitis. Considering the severity of the child’s condition the doctor made the
referral for hospitalization. What diagnosis was most likely stated in the referral?
A. Mucoviscidosis (Cystic fibrosis)*
B. Acute obstructive bronchitis
C. Recurrent obstructive bronchitis
D. Pertussis
E. Acute obstructive pneumonia

51. A 46-year-old man notes swollen legs, weakness, sensation of fullness and heaviness in the
right subcostal area; it is the first occurrence of these signs in the patient. The patient has 20-
year-long history of rheumatoid arthritis. The liver and spleen are enlarged and dense. Blood
creatinine -0,23 mmol/l, proteinemia - 68 g/l, cholesterol - 4,2 mmol/l, urine specific gravity -
1012, proteinuria - 3,3 g/l, isolated wax-like cylinders, leached erythrocytes in the vision field,
leukocytes - 5-6 in the vision field. What is the most likely complication?
A. Renal amyloidosis*
B. Chronic glomerulonephritis
C. Acute glomerulonephritis
D. Heart failure
E. Chronic pyelonephritis

52. A 23-year-old man had taken 1 g of aspirin to treat acute respiratory infection. After that he
developed an asthmatic fit with labored expiration that was arrested by introduction of
aminophylline. The patient has no medical history of allergies. The patient has undergone two

12
Завантажено з сайту https://тестування.укр/ - онлайн тестування КРОК

surgeries for nasal polyposis in the past. What is the most likely diagnosis?
A. Aspirin-induced asthma*
B. Atopic bronchial asthma
C. Infectious allergic bronchial asthma
D. Exercise-induced asthma
E. Symptomatic bronchospasm

53. A 50-year-old patient was delivered to a hospital with complaints of blood traces in urine.
Urination is painless and undisturbed. Macrohematuria had been observed for 3 days.
Objectively: kidneys cannot be palpated, suprapubic area is without alterations, external
genitalia are non-pathologic. On rectal investigation: prostate is not enlarged, painless, has
normal structure. Cystoscopy revealed no alterations. What is the most likely diagnosis?
A. Renal carcinoma*
B. Bladder tuberculosis
C. Varicocele
D. Dystopic kidney
E. Necrotic papillitis

54. A man complains of constant dull pain in the perineum and suprapubic area, weak flow of
urine, frequent difficult painful urination, nocturia. The patient has been suffering from this
condition for several months, during which urination was becoming increasingly difficult, and
pain in the perineum has developed. On rectal examination: the prostate is enlarged (mainly
its right lobe), dense, asymmetrical, central fissure is smoothed out, the right lobe is of stony
density, painless, tuberous. What disease is it?
A. Prostate cancer*
B. Prostate sclerosis
C. Urolithiasis, prostatolith of the right lobe
D. Prostate tuberculosis
E. Chronic congestion prostatitis

55. A boy was born at 32 weeks of gestation. 2 hours after the birth he developed respiratory
distress (RD). The RD severity assessed by Silverman score was 5. The respiratory disorders
progressed, respiratory failure could not be eliminated by Martin-Bouyer CPAP (continuous
positive airway pressure). X-ray of lungs shows reticular and nodular pattern, air
bronchogram. What is the most likely cause of respiratory distress syndrome?
A. Hyaline membrane disease*
B. Segmental atelectasis
C. Bronchopulmonary dysplasia
D. Congenital pulmonary emphysema
E. Edematous hemorrhagic syndrome

56. An 8-year-old child with a 3-year-long history of diabetes was hospitalized in hyperglycemic
coma. Specify the initial dose of insulin to be administered:
A. 0,1-0,2 U/kg of body weight per hour*
B. 0,05 U/kg of body weight per hour
C. 0,2-0,3 U/kg of body weight per hour
D. 0,3-0,4 U/kg of body weight per hour

13
Завантажено з сайту https://тестування.укр/ - онлайн тестування КРОК

E. 0,4-0,5 U/kg of body weight per hour

57. A 17-year-old young man complains of general weakness, trismus, twitching of the muscles in
his left shin. 7 days ago he pierced his foot with a nail. Objectively: at the sole of the foot there
is a wound, 0,3x0,2 mm in size, with small amount of serous-purulent discharge, the skin
around the wound is hyperemic. What is the most likely diagnosis?
A. Tetanus*
B. Phlegmon
C. Osteomyelitis
D. Infected wound
E. Erysipelas

58. A patient with trauma of the lower third of the forearm volar surface caused by a glass shard
came to a first-aid center. Objectively: flexion of the IV and V fingers is impaired, sensitivity of
the inner dorsal and palmar surfaces of the hand and IV finger is decreased. What nerve is
damaged?
A. Ulnar*
B. Radial
C. Median
D. Musculocutaneous
E. Axillary

59. A 58-year-old patient complains of pain in the lower left extremity, which aggravates during
walking, sensation of cold and numbness in the both feet. The patient has been suffering from
this condition for 6 years. Objectively: the skin is pale and dry, with hyperkeratosis. On the left
shin hair is scarce, ”furrow” symptom of inflamed inguinal lymph nodes is positive. Pulse
cannot be detected over the pedal and popliteal arteries and is weakened over the femoral
artery. In the right limb popliteal artery pulsation is retained. What is the most likely
diagnosis?
A. Arteriosclerosis obliterans of the lower extremities*
B. Obliterating endarteritis
C. Femoral artery thrombosis
D. Raynaud’s disease
E. Buerger’s disease (thromboangiitis obliterans)

60. An infant is 2,5 months old. The onset of the disease was gradual, the child had normal body
temperature but presented with slight cough. Within a week the cough intensified, especially
at night; on the 12th day the child developed cough fits occurring up to 20 times per day and
followed by vomiting. There was one instance of respiratory arrest. Make the diagnosis:
A. Pertussis*
B. Parainfluenza
C. Congenital stridor
D. Respiratory syncytial infection
E. Adenovirus infection

61. Vaginal examination reveals the head of the fetus, which fills the posterior surface of

14
Завантажено з сайту https://тестування.укр/ - онлайн тестування КРОК

symphysis pubis and hollow of the sacrum. The lower edge of symphysis pubis, ischiadic
spines, and sacrococcygeal joint can be palpated. Where in the lesser pelvis is the fetal head
situated?
A. In the narrow plane of lesser pelvis*
B. In the wide plane of lesser pelvis
C. Above pelvic inlet
D. In the area of brim
E. In the area of pelvic outlet

62. A man works in casting of nonferrous metals and alloys for 12 years. In the air of working area
there was registered high content of heavy metals, carbon monoxide, and nitrogen. During
periodic health examination the patient presents with asthenovegetative syndrome, sharp
pains in the stomach, constipations, pain in the hepatic area. In urine: aminolevulinic acid and
coproporphyrin are detected. In blood: reticulocytosis, low hemoglobin level. Such intoxication
is caused by:
A. Lead and lead salts*
B. Tin
C. Carbon monoxide
D. Nitric oxide
E. Zinc

63. A 30-year-old multigravida has been in labour for 18 hours. 2 hours ago the pushing stage
began. Fetal heart rate is clear, rhythmic, 136/min. Vaginal examination reveals complete
cervical dilatation, the fetal head in the pelvic outlet plane. Sagittal suture is in line with
obstetric conjugate, the occipital fontanel is near the pubis. The patient has been diagnosed
with primary uterine inertia. What is the further tactics of labour management?
A. Outlet forceps*
B. Labour stimulation
C. Cesarean section
D. Skin-head Ivanov’s forceps
E. Vacuum extraction of the fetus

64. A patient, who had eaten canned mushrooms (honey agaric) three days ago, developed vision
impairment (diplopia, mydriasis), speech disorder, disturbed swallowing. What type of food
poisoning occurred in the patient?
A. Botulism*
B. Food toxicoinfection
C. Fusariotoxicosis
D. Honey agaric poisonong
E. Lead salts poisoning

65. An infant is 3 weeks old. Since birth there has been observed periodical vomiting within a few
minutes after feeding. The amount of vomitive masses does not exceed the volume of previous
feeding. The infant has age-appropriate body weight. What is the most likely cause of this
symptom?
A. Pylorospasm*
B. Esophageal chalasia

15
Завантажено з сайту https://тестування.укр/ - онлайн тестування КРОК

C. Adrenogenital syndrome
D. Pyloristenosis
E. Esophageal achalasia

66. A newborn with gestational age of 31 weeks presents with hypotonia and depressed
consciousness. Hematocrit is 35%, general cerebrospinal fluid analysis shows increased
content of erythrocytes and protein, and low glucose. These data correspond with the clinical
presentation of:
A. Intracranial hemorrhage*
B. Meningitis
C. Sepsis
D. Anemia
E. Prenatal infection

67. From urine of a 14-year-old boy with the exacerbation of secondary obstructive pyelonephritis
Pseudomonas aeruginosa was isolated with a titer of 1000000 microbes per 1 ml. What
antibiotic is the most advisable in this case?
A. Ciprofloxacin*
B. Ampicillin
C. Cefazolinum
D. Azithromycin
E. Chloramphenicol

68. A 20-year-old woman on the 10th day after her discharge from the maternity ward developed
fever up to 39oC and pain in her left mammary gland. On examination the mammary gland is
enlarged, in its upper outer quadrant there is a hyperemic area. In this area a dense spot with
blurred margins can be palpated. The patient presents with lactostasis and no fluctuation.
Lymph nodes in the right axillary crease are enlarged and painful. Specify the correct
diagnosis:
A. Lactational mastitis*
B. Abscess
C. Erysipelas
D. Dermatitis
E. Tumor

69. A 3-year-old girl is being treated at a resuscitation unit with diagnosis ”acute kidney failure,
oligoanuric stage”. ECG: high T wave, extended QRS complex, displacement of S-T interval
downwards below the isoline. What electrolyte imbalance is it?
A. Hyperkalemia*
B. Hypokalemia
C. Hypocalcemia
D. Hypercalcemia
E. Hyperphosphatemia

70. Mother of an 8-year-old girl complains that the child is too short and has excessive body
weight. Objectively: obesity with fat deposits on the torso and face (round moonlike face),

16
Завантажено з сайту https://тестування.укр/ - онлайн тестування КРОК

acne, striae on the thighs and lower abdomen, hirsutism. What hormone can cause such
symptoms, when in excess?
A. Cortisol*
B. Thyroxin
C. Testosterone
D. Insulin
E. Glucagon

71. A 9-year-old girl complains of fever up to 37,5°C, headache, inertness, weakness, loss of
appetite, stomachache, and frequent painful urination. Provisional diagnosis of acute
pyelonephritis is made. Clinical urine analysis: specific gravity - 1018, no protein, leukocytes -
10-15 in the vision field. What investigation method can verify the diagnosis of urinary system
infection?
A. Bacteriological inoculation of urine*
B. Rehberg test (creatinine clearance test)
C. Zymnytsky test (measurement of daily diuresis)
D. Complete blood count
E. Clinical urine analyses, dynamic testing

72. During assessment of work conditions at the mercury thermometer manufacture, content of
mercury vapors in the air of working area is revealed to exceed maximum concentration limit.
Specify the main way of mercury penetration into the body:
A. Respiratory organs*
B. Intact skin
C. Damaged skin
D. Gastrointestinal tract
E. Mucous tunics

73. During health assessment of car drivers and police officers on point duty, the physicians
detected carboxyhemoglobin in the blood of the patients, weakened reflex responses,
disturbed activity of a number of enzymes. Revealed professional health disorders are most
likely to be associated with the effect of:
A. Carbon monoxide*
B. Sulfurous anhydride
C. Mental stress
D. Aromatic hydrocarbons
E. Nitric oxide

74. On the 3rd day of life a newborn, who had suffered birth asphyxia, developed hemorrhage
from the umbilical wound. Laboratory analysis reveals hypocoagulati-on, thrombocytopenia,
and hypothrombi-nemia. What is the cause of such clinical developments?
A. Disseminated intravascular coagulation*
B. Hemorrhagic disease of newborn
C. Congenital angiopathy
D. Thrombocytopenic purpura
E. Umbilical vessel trauma

17
Завантажено з сайту https://тестування.укр/ - онлайн тестування КРОК

75. A 46-year-old woman came to a maternity clinic with complaints of moderate blood discharge
from the vagina, which developed after the menstruation delay of 1,5 months. On vaginal
examination: the cervix is clean; the uterus is not enlarged, mobile, painless; appendages
without changes. Make the diagnosis:
A. Dysfunctional uterine bleeding*
B. Adenomyosis
C. Ectopic pregnancy
D. Submucous uterine myoma
E. Cancer of the uterine body

76. A patient has the second and third degree burns of the 15% of the body surface. On the 20th
day after the trauma the patient presents with sharp increase of body temperature, general
weakness, rapid vesicular respiration; facial features are sharpened, BP is 90/50 mm Hg, heart
rate is 112/min. What complication is it?
A. Sepsis*
B. Pneumonia
C. Acute intoxication
D. Purulent bronchitis
E. Anaerobic infection

77. A 30-year-old woman complains of amenorrhea that lasts for 2 years after she has given birth,
loss of hair and body weight. The labor was complicated with hemorrhage caused by uterine
hypotonia. Objectively the patient is of asthenic type, her external genitalia are hypoplastic,
the uterine body is small in size and painless. No uterine appendages can be detected. What is
the most likely diagnosis?
A. Sheehan’s syndrome (postpartum hypopituitarism)*
B. Ovarian amenorrhea
C. Turner’s syndrome
D. Ovarian exhaustion syndrome
E. Galactorrhea-amenorrhea syndrome

78. On laboratory investigation of a pork sample there is 1 dead trichinella detected in 24


sections. This meat should be:
A. Handed over for technical disposal*
B. Allowed for sale with no restrictions
C. Processed and sold through public catering network
D. Processed for boiled sausage production
E. Frozen until the temperature of - 10oC is reached in the deep layers, with subsequent
exposure to cold for 15 days

79. Examination of a 43-year-old man objectively revealed pallor of skin and mucous membranes,
loss of tongue papillae, transverse striation of fingernails, cracks in the mouth corners,
tachycardia. Blood test results: Hb- 90 g/l, anisocytosis, poikilocytosis. The most likely
causative factor of this condition is the inadequate intake of:
A. Iron*
B. Copper
C. Zinc

18
Завантажено з сайту https://тестування.укр/ - онлайн тестування КРОК

D. Magnesium
E. Selene

80. A 52-year-old woman presents with affected mucosa in the mouth angles where fissures,
erosions, and ulcers develop; vertical fissures appear on the lips during their closing
(cheilosis); there are tongue alterations (glossitis), angular stomatitis, seborrheic dermatitis
around the mouth and wings of the nose, and pericorneal injection. The listed symptoms are
characteristic of:
A. P2-hypovitaminosis*
B. Ei-hypovitaminosis
C. C-hypovitaminosis
D. PP-hypovitaminosis
E. A-hypovitaminosis

81. Two days ago a woman fell from the height of 1,5 m. She complains of severe thoracic pain on
the left and dyspnea. Chest X-ray reveals hydropneumothorax on the left with fluid level at the
7th rib and the lung collapsed by 1/3. The 6th-7th ribs are fractured along the scapular line.
Serohemorrhagic fluid was obtained during thoracic puncture. What treatment tactics should
be chosen?
A. Thoracocentesis on the left at the 7th intercostal space*
B. Thoracocentesis at the 2nd intercostal space
C. Repeated puncture of the pleural cavity
D. Intubation of trachea with artificial pulmonary ventilation
E. Alcohol-novocaine block of the fracture area

82. A patient complains of suppuration from the ear and impaired hearing of the left ear, which
have been observed for the past 6 years. The patient had periodical headaches, general
indisposition, fever. Objectively: otoscopy of the external auditory meatus revealed
mucopurulent odorless substance. The eardrum is of normal color, with central perforation.
What is the most likely diagnosis?
A. Chronic mesotympanitis*
B. Otosclerosis
C. Acute otitis media
D. Chronic epitympanitis
E. Chronic sensorineural hearing loss

83. In 2 hours after a traffic accident a 28-year-old man in grave condition was delivered to a
hospital. The patient complains of abdominal pain. He received a blow to the abdomen with
the steering wheel. Objective examination revealed the following: the abdomen does not
participate in respiration, is tense and acutely painful on palpation; the abdominal muscles are
defensively tense, peritoneal irritation signs are positive, hepatic dullness is absent. BP is
90/60 mm Hg, heart rate is 120/min. What further treatment tactics should be chosen?
A. Laparotomy*
B. Laparoscopy
C. Cold to the abdomen
D. Ultrasound investigation
E. Laparocentesis

19
Завантажено з сайту https://тестування.укр/ - онлайн тестування КРОК

84. A 28-year-old woman complains of increased intervals between menstruations, up to 2 months,


and hirsutism. Gynecological examination revealed the following: ovaries are enlarged,
painless, and dense; no alterations of the uterus. US of the lesser pelvis: ovaries are 4-5 cm in
diameter, with numerous enlarged follicles on the periphery. X-ray of the skull base: sellar
region is widened. What is the most likely diagnosis?
A. Stein-Leventhal syndrome (polycystic ovarian syndrome)*
B. Algodismenorrhea
C. Sheehan syndrome (postpartum hypopituitarism)
D. Premenstrual syndrome
E. Morgagni-Stewart-Morel syndrome (metabolic craniopathy)

85. Posture of an 11-year-old boy was determined during preventive examination. The child
presents with curled forward rounded shoulders, the head is bowed forward, the thorax is
flattened, the stomach is bulging. In the vertebral column there are deepened cervical and
lumbar flexures. What posture does the child have?
A. Kyphosis*
B. Lordosis
C. Stooping
D. Corrected
E. Normal

86. A 46-year-old woman has been hospitalized with open fracture of the left thigh in its middle
third. She underwent the surgery - fixation with extraosseous osteosynthesis plates. On the 4th
day after the surgery she developed pain in the wound, body temperature rose over 39oC.
What measures should be taken in this case?
A. Undo the sutures, drain the wound, and prescribe antibiotics*
B. Prescribe broad spectrum antibiotics and hormonal agents
C. Administer antibiotics intraosseously and hypothermia locally
D. Inject antibiotics into the area surrounding the wound, prescribe spasmolytics and
analgesics
E. Remove the fixation, prescribe sulfanilami-des

87. A multigravida on the 38th week of her pregnancy complains of increased BP up to 140/90 mm
Hg, edema of the shins for 2 weeks. In the last month she gained 3,5 kg of weight. Urine
analysis: protein - 0,033 g/l. Make the diagnosis:
A. Mild preeclampsia*
B. Moderate preeclampsia
C. Pregnancy hypertension
D. Severe preeclampsia
E. Pregnancy edema

88. A 44-year-old patient with postinfarction cardiosclerosis presents with frequent heart rate
disorders and lower extremity edema. Objectively: Ps- 95/min., irregular, 10-12 extrasystoles
per minute. BP- 135/90 mm Hg. The 1st heart sound at the apex is weakened. Pulmonary
respiration is rough. The liver is enlarged +2 cm. ECG: irregular sinus rhythm, heart rate -
95/min, frequent polytopic ventricular extrasystoles. What antiarrhythmic drug is advisable in
this case for treatment and prevention of extrasystole?

20
Завантажено з сайту https://тестування.укр/ - онлайн тестування КРОК

A. Amiodarone*
B. Lidocaine
C. Mexiletine
D. Quinidine
E. Novocainamide (Procainamide)

89. A 60 year-old woman has been suffering from weakness, dizziness, and fatigue over the last
year. Recently she has also developed dyspnea, paresthesia. Objectively: the skin and mucous
membranes are pale and slightly icteric. The tongue is smooth due to the loss of lingual
papillae. Liver and spleen are located at the costal margin. Blood count: Hb- 70 g/l, RBC-1,
71012/l, color index -1,2, macrocytes. Administer the patient a pathogenetically justified drug:
A. Vitamin B12*
B. Vitamin B6
C. Ascorbic acid
D. Iron preparations
E. Vitamin B1

90. After excessive consumption of fatty food a 60-year-old woman suddenly developed pain in her
right subcostal area, nausea, bile vomiting, sharp bitter taste in her mouth. In 2 days she
developed jaundice, her urine darkened. Objectively: sclera and skin are icteric, the abdomen
is distended, the liver is enlarged by 3 cm, soft and painful on palpation, Ortner’s, Murphy’s,
Kehr’s, Zakharyin’s, Mayo-Robson’s signs are positive. What diagnostic technique should be
used in the first place?
A. US of the gallbladder and bile duct*
B. Fibrogastroduodenoscopy
C. X-ray of the abdomen
D. Radionuclide scanning of the liver and gallbladder
E. Laparoscopy

91. A 42-year-old woman working at a poultry farm complains of dyspnea, thoracic pain on the
left, increased body temperature up to 38-39oC in the evening, and cough. The patient suffers
from essential hypertension. Objectively: vesicular respiration in the lungs, vesicular
resonance without alterations. X-ray of both lungs reveals numerous small low-intensity foci,
2-3 mm in size, located in a row along the blood vessels. ESR- 32 mm/hour. What is the most
likely diagnosis?
A. Acute disseminated tuberculosis*
B. Infiltrative tuberculosis
C. Pulmonary carcinomatosis
D. Focal tuberculosis
E. Community-acquired pneumonia

92. A 65-year-old patient has been suffering from liver cirrhosis associated with hepatitis C virus
for 7 years. During the last 3 weeks the patient developed severe edema of the lower
extremities, the abdomen is significantly distended with fluid. Ultrasound: signs of liver
cirrhosis, portal hypertension, lymph nodes are in the area of hepatic portal. Concentration of
a-fetoprotein in blood serum is 285 ng/ml. What complication could have developed in this
case?

21
Завантажено з сайту https://тестування.укр/ - онлайн тестування КРОК

A. Hepatocellular carcinoma of the liver*


B. Hepatocellular insufficiency
C. Hypersplenism
D. Cholestasis
E. Congestive heart failure

93. A 43-year-old man has undergone a surgery for osteomyelitis of the left thigh. On the 6th day
the patient’s condition was complicated with sepsis. Despite complex therapy of sepsis on the
9th day the patient persistently presents with temperature up to 40oC, heart rate is 110/min.,
respiration rate is 23/min., BP is 100/60 mm Hg. Blood test: leukocytes -16 • 109/l, band
neutrophils -16%. What phase of clinical course is it?
A. Catabolic*
B. Anabolic
C. Rehabilitation
D. Functional
E. Tension

94. A 23-year-old woman presents with diabetes mellitus type 1. She complains of weakness,
headache, nausea, and vomiting. Objectively: temperature is 37,6°C, heart rate is 98/min., BP
is 95/65 mm Hg, respiration rate is 32/min., loud. Smell of acetone is detected, heart sounds
are muffled, pulse is rhythmic. The stomach is sensitive in the epigastrium. Costovertebral
angle tenderness (Murphy’s punch sign) is present. Blood glucose is 28,5 mmol/l; blood
leukocytes - 16, 5 • 109/l. In urine: acetone ++, leukocytes - 25-40 in the vision field. Blood pH
is 7,1. What correction method would be the most advisable?
A. Infusion of normal saline + insulinotherapy in small portions*
B. Infusion of 5% glucose + insulinotherapy in large portions
C. Infusion of 2,5% sodium bicarbonate + insulinotherapy
D. Intravenous administration of antibiotics + insulinotherapy
E. Infusion of dextran solutions + insuli-notherapy

95. A 57-year-old woman complains of a sensation of esophageal compression, palpitations,


difficult breathing when eating solid food, occasional vomiting with a full mouth, ”wet pillow”
sign at night for the last 6 months. Objectively: body tempearture -39oC, height - 168 cm,
weight - 72 kg, Ps-76/min, BP- 120/80 mm Hg. X-ray study revealed considerable dilation of
esophagus and its constriction in the cardial part. What pathology is the most likely to have
caused dysphagia in this patient?
A. Achalasia cardiae*
B. Primary esophageal spasm
C. Hiatal hernia
D. Esophageal carcinoma
E. Reflux esophagitis

96. A 47-year-old man is employed at the weaving workshop, has 15-year-long record of service at
this factory; his work conditions are associated with high-frequency and high-intensity noise.
During periodical examination he was diagnosed with occupational deafness. What are the
grounds for making such a diagnosis?
A. Audiometry data and hygienic assessment of working environment*

22
Завантажено з сайту https://тестування.укр/ - онлайн тестування КРОК

B. Record of service at this factory


C. Noise characteristic at this factory
D. Central nervous system examination results
E. Inner ear examination results

97. A 37-year-old woman complains of acute pain in the genital area, swelling of the labia, pain
when walking. Objectively: body temperature is 38,7oC, Ps- 98/min. In the interior of the right
labia there is a dense, painful tumor-like formation 5,0x4,5 cm in size, the skin and mucous
membrane of genitals are hyperemic, there is profuse foul-smelling discharge. What is the
most likely diagnosis?
A. Acute bartholinitis*
B. Labial furuncle
C. Acute vulvovaginitis
D. Bartholin gland cyst
E. Carcinoma of vulva

98. A woman undergoing in-patient treatment for viral hepatitis type B developed headache,
nausea, recurrent vomiting, memory lapses, flapping tremor of her hands, rapid pulse. Sweet
smell from the mouth is detected. Body temperature is 37,6°C, heart rate is 89/min. What
complication developed in the patient?
A. Acute liver failure*
B. Ischemic stroke
C. Gastrointestinal hemorrhage
D. Hypoglycemic shock
E. Meningoencephalitis

99. A 42-year-old man was delivered to a surgical in-patient department with complaints of icteric
skin, pain in the right subcostal area. Biochemical blood analysis: total bilirubin - 140 mcmol/l,
direct bilirubin - 112 mcmol/l. On US: choledoch duct - 1,4 cm, a concrement is detected in the
distal area. Gallbladder is 40 cm, no concrements. What treatment tactics should be chosen?
A. Endoscopic papillosphincterotomy*
B. Laparoscopic cholecystectomy
C. Laparotomy with choledoch duct drain
D. Laparotomy with cholecystectomy
E. Threatment in an infectious diseases hospital

100. 4 weeks after myocardial infarction a 56-year-old patient developed acute heart pain, marked
dyspnea. Objectively: the patient’s condition is extremely grave, there is marked cyanosis of
face, swelling and throbbing of neck veins, peripheral pulse is absent, the carotid artery pulse
is rhythmic, 130/min., BP is 60/20 mm Hg. Auscultation of the heart reveals extremely muffled
sounds, percussion reveals heart border extension in both directions. What is the optimal
treatment tactics for this patient?
A. Pericardiocentesis and immediate thoracotomy*
B. Oxygen inhalation
C. Puncture of the pleural cavity on the left
D. Conservative treatment, infusion of adrenomimetics
E. Pleural cavity drainage

23
Завантажено з сайту https://тестування.укр/ - онлайн тестування КРОК

101. A patient with chronic pancreatitis complains of diarrhea occurring up to 5 times per day (no
blood traces), loss of body weight, abdominal distention, dryness of skin, loss of hair, thirst,
bleeding gums, convulsions. Complete blood count: leukocytes -5, 8 • 109/l; Hb- 86 g/l; ESR-
15 mm/g; Blood protein test: protein - 48 g/l; albumins - 28 g/l. What indicators of coprological
analysis would accompany this syndrom?
A. Steatorrhea, creatorrhea*
B. Large amount of mucus, amylorrhea
C. Large amount of starch grains and cellulose
D. Gas bubbles, acid reaction
E. Large numbers of iodinophilous microbes

102. A 35-year-old patient complains of heartburn, gasseous and sour eructation, burning
constricting pain behind the sternum and along the esophagus, developing when bowing the
torso to the front. No previous examination; the patient takes almagel at his own discretion,
after which he notes improvement of his general state. Make the provisional diagnosis:
A. Gastroesophageal reflux disease*
B. Functional dyspepsia
C. Cardiospasm
D. Ulcer disease of the stomach
E. Ulcer disease of the duodenum

103. The left hand of a newborn is extended in all its joints, stretched along the torso, and pronated
in the forearm. Active movements of the shoulder joint are retained. The hand is flattened,
atrophied, cold to touch, hangs passively. Grasping and Babkin’s reflexes are absent at the
affected side. Hemogram indicators are normal. Make the most likely diagnosis:
A. Inferior distal obstetrical paralysis*
B. Osteomyelitis
C. Proximal obstetrical paralysis
D. Complete obstetrical paralysis
E. Hypoxic-ischemic encephalopathy

104. A 34-year-old man is being treated for schizophrenia exacerbation in a psychiatric unit.
Objectively: the patient remains in bed, is sluggishly mobile, unresponsive, does not react to
questions. His position is unvaried, hypomimic, snout reflex and Dupre’s symptom are present,
muscles exhibit waxy flexibility. He has been remaining in this state for approximately a week.
Feeding is parenteral. Determine the neuromotor disturbance:
A. Catatonic stupor*
B. Depressive stupor
C. Psychogenic stupor
D. Anergic stupor
E. Exogenic stupor

105. A 14-year-old girl has been delivered to a gynecological department with complaints of profuse
blood discharge from her genital tract for 2 weeks. Anamnesis: menstruation since 13,
irregular, painful, profuse; the last one was 2 months ago. Objectively: pale skin and mucosa,
BP- 100/60 mm Hg, Hb-108 g/l. The abdomen is soft and painless on palpation. Rectal
examination revealed no pathologies of reproductive organs. What condition is it?

24
Завантажено з сайту https://тестування.укр/ - онлайн тестування КРОК

A. Juvenile uterine hemorrhage (Dysfunctional)*


B. Hypomenstrual syndrome
C. Inflammation of uterine appendages (Pelvic inflammatory disease)
D. Pelviperitonitis
E. Endometritis

106. An infant cries during urination, the foreskin swells and urine is excreted in drops. What
approach to treatment should be chosen?
A. Create an opening into the foreskin cavity*
B. Prescription of a-adrenergic blocking agents
C. Prescription of antispasmodic agents
D. Urinary bladder catheterization
E. Epicystostomy

107. A 20-year-old patient complains of pain in the left lumbar region, arterial pressure rise up to
160/110 mm Hg. US revealed that the structure and size of the right kidney were within age
norms, there were signs of the 3rd degree hydronephrotic transformation of the left kidney.
Doppler examination revealed an additional artery running to the lower pole of the kidney.
Excretory urogram shows a narrowing in the region of ureteropelvic junction. Specify the
treatment tactics:
A. Surgical intervention*
B. Administration of spasmolytics
C. Administration of ACE inhibitors
D. Kidney catheterization
E. Administration of ^-blockers

108. A 49-year-old patient consulted a doctor about difficult swallowing, voice hoarseness, weight
loss. These symptoms have been gradually progressing for the last 3 months. Objectively: the
patient is exhausted, there are enlarged supraclavicular lymph nodes. Esophagoscopy revealed
no esophageal pathology. Which of the following investigations is the most appropriate in this
case?
A. Computed tomography of chest and mediastinum*
B. X-ray of lungs
C. Multiplanar imaging of esophagus
D. Radioisotope investigation of chest
E. Ultrasound investigation of mediastinum

109. A 15-year-old girl complains of dizziness and sensation of lack of air that she develops in
emotionally straining situations. Relief occurs after she takes corvalol. Objectively:
hyperhidrosis and marble-like pattern of the skin of her palms and feet. Clinical and
instrumental examination revealed no organic alterations of the central nervous,
cardiovascular, and respiratory systems. What provisional diagnosis can be made?
A. Somatoform autonomic dysfunction*
B. Obstructive bronchitis
C. Bronchial asthma
D. Stenosing laryngotracheitis
E. Acute epiglottitis

25
Завантажено з сайту https://тестування.укр/ - онлайн тестування КРОК

110. A 14-year-old patient complains of alopecia foci on his scalp. The patient has been presenting
with this condition for 2 weeks. Objectively: on the scalp there are several small oval foci with
blurred margins. The skin in the foci is pink-red, the hairs are broken off at 4-5 mm length or
at skin level. Under Wood’s lamp there are no foci of green luminescence detected. What
disease is it?
A. Trichophytosis capitis*
B. Syphilitic alopecia
C. Alopecia areata
D. Scleroderma
E. Psoriasis

111. A 38-year-old patient has been delivered by an ambulance to a surgical department with
complaints of general weakness, indisposition, black stool. On examination the patient is pale,
there are dotted hemorrhages on the skin of his torso and extremities. On digital investigation
there are black feces on the glove. Blood test: Hb- 108 g/l, thrombocytopenia. Anamnesis
states that similar condition was observed 1 year ago. Make the diagnosis:
A. Thrombocytopenic purpura*
B. Hemophilia
C. Ulcerative bleeding
D. Rectal tumor
E. Nonspecific ulcerative colitis

112. A 35-year-old patient developed an epileptic attack with tonoclonic spasms that lasted for 3
minutes. After the attack the patient fell asleep but in 5 minutes the second attack occurred.
The first step of emergency aid would be to:
A. Ensure patency of airways*
B. Take blood from the vein for analysis
C. Introduce diazepam intravenously
D. Prescribe antiepileptic drugs
E. Administer chloral hydrate via an enema

113. A 23-year-old woman has been suffering from a mental disease since the age of 18, the course
of disease has no remission periods. At a hospital the patient mostly presents with non-
purposeful foolish excitation: she makes stereotypic grimaces, exposes herself, publicly
masturbates with a loud laughter, repeates stereotypical abusive shouts. The patient should be
prescribed:
A. Neuroleptics*
B. Antidepressants
C. Tranquilizers
D. Nootropics
E. Mood stabilizers

114. A young woman suffering from seborrhea oleosa has numerous light-brown and white spots on
the skin of her torso and shoulders. The spots have clear margins, branny desquamation, no
itching. What provisional diagnosis can be made?
A. Pityriasis versicolor*
B. Torso dermatophytosis

26
Завантажено з сайту https://тестування.укр/ - онлайн тестування КРОК

C. Seborrheic dermatitis
D. Pityriasis rosea
E. Vitiligo

115. 10 days after birth a newborn developed sudden fever up to 38,1oC. Objectively: the skin in
the region of navel, abdomen and chest is erythematous; there are multiple peasized blisters
with no infiltration at the base; isolated bright red moist erosions with epidermal fragments
are observed on the periphery. What is the provisional diagnosis?
A. Epidemic pemphigus of newborn*
B. Syphilitic pemphigus
C. Streptococcal impetigo
D. Vulgar impetigo
E. Atopic dermatitis

116. A man came to an urologist with complains of painful urination, discharge from urethra. The
patient has been suffering from this condition for a week. Objectively: hyperemic urinary
meatus, edema, purulent discharge. Microscopy of smears detected gram-negative bacteria.
Specify the diagnosis:
A. Acute gonorrheal urethritis*
B. Trichomonas urethritis
C. Candidal urethritis
D. Chlamydial urethritis
E. Chronic gonorrhea

117. A patient is 28 years old. He has been suffering from mental disorder since he was 22. His
current condition has changed acutely: for 3 days the patient has been refusing to leave his
home. He claims that there is a ’’telepathy” occurring between him and the other people,
through which he receives ’thoughts of strangers” and transmits his own thoughts for
everyone to hear. He is convinced that his thoughts and actions are being manipulated
through this ’telepathy’.’ Make the preliminary diagnosis:
A. Paranoid schizophrenia*
B. Depressive episode
C. Manic episode
D. Organic delirium
E. Acute reaction to stress

118. A 35-year-old woman has gained 20 kg weight within a year with the normal diet. She
complains of chills, sleepiness, shortness of breath. The patient’s mother and sister are
corpulent. Objectively: height - 160 cm, weight - 92 kg, BMI- 35,9. Obesity is uniform, there
are no striae. The face is amimic. The skin is dry. The tongue is thickened. Heart sounds are
muffled. HR- 56/min, BP-140/100 mm Hg. The patient has been suffering from amenorrhea for
5 months, has constipations. TSH- 28 mclU/l (norm is 0,32-5). Craniogram shows no pathology.
What is the etiology of obesity?
A. Hypothyroid*
B. Hypo-ovarian
C. Hypothalamic-pituitary
D. Alimentary and constitutive

27
Завантажено з сайту https://тестування.укр/ - онлайн тестування КРОК

E. Hypercorticoid

119. A patient complains of painless ’sores” on his penis and inguinal lymph nodes enlargement.
Synthomycin emulsion that the patient have been applying to the ’sores” was ineffective.
Objectively: on the inner leaf of the foreskin there are three closely situated rounded erosions,
0,5 cm in diameter, with dense infiltration that can be palpated at their bases. Make the
preliminary diagnosis:
A. Primary syphilis*
B. Herpes simplex (Herpes pro genitalis)
C. Candidiasis of the inner leaf of the foreskin
D. Shingles
E. Erythema multiforme

120. A 1,5-month-old child on breasfeedi-ng presents from birth with daily vomiting, irregular liquid
foamy feces, and meteori-sm, which are resistant to antibacterial and probiotic therapy; no
increase of body mass is observed. The child’s condition improved, when breastmilk was
substituted with ”NAN low lactose” formula. What pathology is it?
A. Lactase deficiency*
B. Intestinal lambliasis (Giardiasis)
C. Infectious enteritis
D. Drug-induced enteritis
E. Functional dyspepsia

121. A 12-year-old girl after a case of respiratory infection developed dyspnea at rest, paleness of
skin. Heart rate is 110/min., BP is 90/55 mm Hg. Heart sounds are muffled. Borders of relative
heart dullness: right - the parasternal line, upper - the III rib, left -1,0 cm outwards from the
midclavicular line. Make the provisional diagnosis:
A. Infectious myocarditis*
B. Functional cardiopathy
C. Somatoform autonomic dysfunction
D. Hypertrophic cardiomyopathy
E. Exudative pericarditis

122. A 48-year-old man complains of fatigue, excessive sweating, severe skin itching, undulant
fever, enlarged cervical and supraclavicular lymph nodes. Objectively: paleness of skin and
mucosa, cervical lymph nodes are mobile, dense, elastic, walnut-sized, painless, not attached
to the skin. Complete blood count: erythrocytes - 3,0 • 1012/l, Hb-100 g/l, leukocytes - 14 •
109/l, eosinophils -6%, basophils - 3%, band neutrophils -11%, segmented neutrophils - 69%,
lymphocytes -7, monocytes - 4%, platelets - 280 • 109/l, ESR-37 mm/hour. What method should
be applied to verify the diagnosis?
A. Lymph node biopsy*
B. Sternal puncture
C. Muscle biopsy
D. Chest X-ray
E. Lumbar puncture

28
Завантажено з сайту https://тестування.укр/ - онлайн тестування КРОК

123. A patient after a blow to the head developed general symptoms of cerebral disturbance,
nausea, vomiting, focal signs -hemi-hyperreflexia S>D, hemihyperesthesia on the left, marked
meningeal syndrome. Neither cranial X-ray nor computer tomography revealed any
pathologies. What examination method would allow making and clarification of the diagnosis?
A. Lumbar puncture*
B. Echoencephalography
C. Electroencephalography
D. Angiography
E. Pneumoencephalography

124. A 72-year-old man complains of lower extremity edema, sensation of heaviness in the right
subcostal area, dyspnea of rest. For over 25 years he has been suffering from COPD.
Objectively: orthopnea, jugular venous distention, diffuse cyanosis, acrocyanosis. Barrel chest
is observed, on percussion there is vesiculotympanitic (bandbox) resonance, sharply weakened
vesicular respiration on both sides, moist crepitant crackles in the lower segments of the
lungs. Heart sounds are weakened, the II heart sound is accentuated over the pulmonary
artery. The liver is +3 cm. What complicated the clinical course of COPD in this patient?
A. Chronic pulmonary heart*
B. Pulmonary embolism
C. Acute left ventricular failure
D. Diffuse pneumosclerosis
E. Community-acquired pneumonia

125. A multigravida at 39 weeks of gestation presenting with regular labour activity for 8 hours has
been delivered to a hospital; the waters broke an hour ago. She complains of headache, seeing
spots. BP is 180/100 mm Hg. Urine test results: protein - 3,3 g/l, hyaline cylinders. Fetal heart
rate is 140/min, rhythmical. Vaginal examination reveals complete cervical dilatation, the fetal
head is on the pelvic floor, sagittal suture is in line with obstetric conjugate, the occipital
fontanel is under the pubis. What is the optimal tactics of labour management?
A. Outlet forceps*
B. Cavity forceps
C. Cesarean section
D. Vacuum extraction of the fetus
E. Conservative labour management

126. After the contact with chemicals a plant worker has suddenly developed stridor, voice
hoarseness, barking cough, progressing dyspnea. Objective examination reveals acrocyanosis.
What is the provisional diagnosis?
A. Laryngeal edema*
B. Laryngeal carcinoma
C. PATE
D. Pulmonary atelectasis
E. Pneumothorax

127. An 18-year-old woman complains of pain in her lower abdomen, profuse purulent discharge
from the vagina, temperature rise up to 37,8oC. Anamnesis states that she had a random
sexual contact the day before the signs appeared. She was diagnosed with acute bilateral

29
Завантажено з сайту https://тестування.укр/ - онлайн тестування КРОК

adnexitis. On additional examination: leukocytes are present throughout all vision field,
bacteria, diplococci with intracellular and extracellular position. What is the most likely agent
in the given case?
A. Neisseria gonorrhoeae*
B. Escherichia coli
C. Chlamydia trachomatis
D. Trichomona vaginalis
E. Staphylococcus aureus

128. A 58-year-old man complains of general weakness, loss of 10 kg of weight within 1,5 months,
progressive pain in the lumbar region, increased blood pressure up to 220/160 mm Hg, low
grade fever. Objectively: in the right hypochondrium deep palpation reveals a formation with
uneven surface and low mobility; veins of the spermatic cord and scrotum are dilated. Blood
test results: Hb- 86 g/l, ESR- 44 mm/h. Urine test results: specific gravity - 1020, protein - 0,99
g/l, RBC cover the whole field of vision, WBC- 4-6 in the field of vision. What is the provisional
diagnosis?
A. Renal tumour*
B. Urolithiasis
C. Acute pyelonephritis
D. Acute glomerulonephritis
E. Nephroptosis

129. Anamnesis of a 30-year-old patient includes closed thoracic injury. Lately the patient has been
suffering from increasing dyspnea, sensation of heaviness in the right subcostal area, and
heart rate disturbances. Objectively: acrocyanosis, bulging cervical veins, ascites, edema of
the lower extremities. Heart auscultation reveals muffled heart sounds, additional III heart
sound is detected. Provisional diagnosis of constrictive pericarditis was made. What diagnostic
technique would NOT confirm the diagnosis?
A. US of abdomen*
B. Computer tomography
C. Echocardiography
D. Magnetic resonance imaging
E. Chest X-ray

130. A 72-year-old woman suffers from diabetes mellitus type 2, concomitant diseases are stage 2
hypertension and stage 2B heart failure. She takes metformin. Hypertensic crisis had occurred
the day before, after which the patient developed extreme weakness, myalgias, thirst, dry
mouth, polyuria. BP is 140/95 mm Hg, heart rate is 98/min., no edemas or smell of acetone
detected. What measures should be taken to prevent development of comatose state in the
patient?
A. Stop metformin, prescribe short-acting insulin*
B. Double the dosage of metformin
C. Apply hypotonic solution of sodium chloride
D. Additionally prescribe long-acting insulin
E. Prescribe glibenclamide

131. The body of a 24-year-old woman with probable signs of poisoning has been found on the

30
Завантажено з сайту https://тестування.укр/ - онлайн тестування КРОК

street. Forensic medical examination was requested by an investigator during examination of


the site and the body. According to the Criminal Procedure Code currently in force in Ukraine,
forensic medical examination is required when it is necessary to determine the:
A. Cause of death*
B. Manner of death
C. Time of death
D. Mode of death
E. Mechanism of death

132. It is the 3rd day after the normal term labor; the infant is rooming-in with the mother and is on
breastfeeding. Objectively: the mother’s general condition is satisfactory. Temperature is
36,4oC, heart rate is 80/min., BP is 120/80 mm Hg. Mammary glands are soft and painless;
lactation is moderate, unrestricted milk flow. The uterus is dense, the uterine fundus is located
by 3 fingers width below the navel. Lochia are sanguino-serous, moderate in volume. Assess
the dynamics of uterine involution:
A. Physiological involution*
B. Subinvolution
C. Lochiometra
D. Pathologic involution
E. Hematometra

133. Survey radiograph of a 52-year-old worker of an agglomeration plant (28-year-long record of


service, the concentration of metal dust is 22-37 mg/m3) shows mildly pronounced interstitial
fibrosis with diffused contrasting well-defined small nodular shadows. The patient has no
complaints. Pulmonary function is not compromised. What is the provisional diagnosis?
A. Siderosis*
B. Silicosis
C. Anthraco-silicatosis
D. Silicatosis
E. Anthracosis

134. A 53-year-old woman complains of weight loss up to 10 kg within the last 2 years, liquid foul-
smelling stool two times a day that poorly washes off the toilet, periodic bouts of nausea,
girdle pain in the upper abdomen. Objectively: pain in Gubergrits zone (on the right from
navel) and at Mayo-Robson’s point. Biochemical blood analysis: glucose - 3,2 mmol/l, bilirubin -
16,5 mcmol/l, crude protein - 56,4 g/l. Urine diastase/amylase - 426 g/h/l. D-xylose test (oral
administration of 25 g of d-xylose) after 5 hours reveals 3 g of xylose in urine. The most likely
diagnosis is:
A. Pancreatitis. Malabsorption syndrome*
B. Pseudomembranous colitis
C. Nonspecific ulcerative colitis
D. Irritable bowel syndrome
E. Chronic gastritis

135. A 15-year-old teenager has undergone medical examination in military recruitment center. The
following was revealed: interval systolic murmur at the cardiac apex, accent of the II heart
sound over the pulmonary artery, tachycardia. What additional examination method will be the

31
Завантажено з сайту https://тестування.укр/ - онлайн тестування КРОК

most informative for determining diagnosis?


A. Echocardiography*
B. Electrocardiography
C. X-ray
D. Phonocardiography
E. Rheography

136. A 64-year-old patient has been hospitalized with complaints of progressive jaundice that
developed over 3 weeks without pain syndrome and is accompanied by general weakness and
loss of appetite. Objectively: temperature is 36,8oC, heart rate is 78/min, abdomen is soft and
painless, peritoneum irritation symptoms are not detected, palpation reveals sharply enlarged
tense gallbladder. What disease can be characterised by these symptoms?
A. Cancer of pancreas head*
B. Duodenal ulcer
C. Acute cholecystitis
D. Chronic cholecystitis
E. Cholecystitis caused by lambliasis

137. A 6-year-old girl came to a general practitioner with her mother. The child complains of
burning pain and itching in her external genitalia. The girl was taking antibiotics the day
before due to her suffering from acute bronchitis. On examination: external genitalia are
swollen, hyperemic, there is white deposit accumulated in the folds. The most likely diagnosis
is:
A. Candidal vulvovaginitis*
B. Trichomoniasis
C. Nonspecific vulvitis
D. Helminthic invasion
E. Herpes vulvitis

138. A 37-year-old patient complains of pain in the spinal column, reduced mobility. The condition
persists for 7 years. ”Sway back” is observed, there is no movement in all spinal regions. On X-
ray: ”bamboo spine” is detected. What is the most likely diagnosis?
A. Ankylosing spondylitis*
B. Osteochondrosis
C. Spondylitis deformans
D. Tuberculous spondylitis
E. Spondylolisthesis

139. A 25-year-old patient is not married and has sexual relations with several partners. During the
last 3 months he noticed a small amount of mucoserous discharge from the urethra.
Subjectively: periodical itching or burning pain in the urethra. Two months ago pain in the
knee joint developed. Possibility of trauma or exposure to cold is denied by the patient. During
the last week eye discomfort is noted - lacrimation and itching. What provisional diagnosis can
be made?
A. Reactive arthritis*
B. Rheumatoid arthritis
C. Seasonal pollinosis

32
Завантажено з сайту https://тестування.укр/ - онлайн тестування КРОК

D. Bacterial nonspecific urethral conjunctivitis


E. Upper respiratory tract infection (URTI) that affects conjunctiva and joints

140. A 19-year-old woman complains of severe pain in the axillary crease. Condition onset occurred
a week ago after her swimming in a cold river and epilation. The next day a painful ”boil”
appeared. The ”boil” was increasing in size every day and became a plum-sized tumor. Upon
examination there are nodular conical growths joined together detected, the skin covering
them is bluish-red in color. Some nodules have fistulous openings producing thick purulent
mass. Body temperature is 38, 5oC, general malaise. What is the most likely diagnosis?
A. Hydradenitis*
B. Carbuncle
C. Cutaneous tuberculosis
D. Necrotizing ulcerative trichophytosis
E. Pyoderma chancriformis

141. A woman complains of weight gain, chills, edema, xeroderma, somnolence, difficulties with
focusing. Objectively: height is 165 cm; weight is 90 kg; body proportions are of female type,
to- 35,8oC, heart rate -58/min, BP- 105/60 mm Hg. Heart sounds are weakened, bradycardia is
observed. Other internal organs have no alterations. Thyroid gland cannot be palpated. Milk
secretion from mammary glands is observed. Hormone test revealed increased levels of
thyroid-stimulating hormone (TSH) and prolactin, and decreased level of thyroxine (T4). What
is the cause of obesity?
A. Primary hypothyroidism*
B. Secondary hypothyroidism
C. Prolactinoma
D. Hypopituitarism
E. Adiposogenital dystrophy

142. ECG revealed the following in a 10-year-old child: sharp acceleration of the heart rate -
240/min., P wave overlaps with T wave and deforms it, moderate lengthening of PQ interval,
QRS complex is without alterations. What pathology does this child have?
A. Paroxysmal atrial tachycardia*
B. Atrial hypertrophy
C. Ventricular hypertrophy
D. WPW syndrome
E. Extrasystole

143. A 54-year-old patient complains of weakness, jaundice, itching skin. Disease onset was 1,5
months ago: fever up to 39oC appeared at first, with progressive jaundice developed 2 weeks
later. On hospitalization jaundice was severely progressed. Liver cannot be palpated.
Gallbladder is enlarged and painless. Blood bilirubin is 190 mcmol/l (accounting for direct
bilirubin). Stool is acholic. What is the most likely jaundice genesis in this patient?
A. Mechanical jaundice*
B. Hepatocellular jaundice
C. Hemolytic jaundice
D. Caroli syndrome
E. Gilbert’s syndrome

33
Завантажено з сайту https://тестування.укр/ - онлайн тестування КРОК

144. During hemotransfusion the patient developed nausea, tremor, lumbar and retrosternal pain.
On examination the skin is hyperemic, later developed pallor; the patient presents with
hyperhidrosis, labored respiration, pulse is 110/min., BP is 70/40 mm Hg. Urine is black
colored. What complication developed in the patient?
A. Posttransfusion shock*
B. Acute renal failure
C. Pulmonary embolism
D. Anaphylactic shock
E. Hypotonic crisis

145. A 22-year-old man suddenly developed extreme weakness, nausea, vomiting with traces of
blood. The patient is known to suffer from peptiv ulcer disease of duodenum and hemophilia A.
Objectively: heart rate -102/min., BP-100/60 mm Hg. Complete blood count: erythrocytes - 3,2
• 1012/l, Hb- 98 g/l, color index - 0,92, leukocytes - 7,4 • 109/l, platelets - 240 • 109/l, ESR-11
mm/hour. What measure would most effectively decrease hemorrhaging in this case?
A. Cryoprecipitate*
B. Aminocapronic acid
C. Native plasma
D. Direct transfusion of donor blood
E. Platelet concentrate transfusion

146. A 22-year-old woman complains of amenorrhea for 8 months. Anamnesis states that menarche
occured at the age of 12,5. Since the age of 18 the patient has a history of irregular
menstruation. The patient is nulli-gravida. The mammary glands are developed properly,
nipples discharge drops of milk when pressed. Hormone test: prolactin level is 2 times higher
than normal. CT reveals a bulky formation with diameter of 4 mm in the region of sella. What
is the most likely diagnosis?
A. Pituitary tumour*
B. Lactational amenorrhea
C. Stein-Leventhal syndrome (polycystic ovary syndrome)
D. Sheehan’s syndrome (postpartum hypopituitarism)
E. Cushing’s disease

147. A 13-year-old girl complains of fatigability, frequent headaches, cardialgia. Eight years ago she
had a case of pyelonephritis. Urine analyses periodically revealed leukocyturia. The child has
undergone no further treatment. On examination: increased BP up to 150/10 0 mm Hg.
Ultrasound investigation revealed significant reduction of the right kidney. What process is
leading in arterial hypertension pathogenesis in this case?
A. Hyperactivity of renin-angiotensin system*
B. Disturbance of water-electrolytic balance
C. Disturbance of renal circulation
D. Hypersympathicotonia
E. Increased cortisol level

148. A 3-month-old child presents with saffron-yellow coloring of the skin, sclera, and mucous
membranes. The abdomen is enlarged, hepatomegaly and splenomegaly are observed. In blood
there is conjugated bilirubin-induced hyperbilirubinemia. On intravenous

34
Завантажено з сайту https://тестування.укр/ - онлайн тестування КРОК

cholangiocholecystography: opacified bile is discharged into the intestine. Transaminase


activity is normal. What is the most likely diagnosis?
A. Biliary atresia*
B. Physiologic jaundice
C. Hemolytic disease of newborn
D. Crigler-Najjar syndrome
E. Congenital hepatitis

149. A 51-year-old woman complains of headache, trembling, paresthesiae, palpitations, increased


blood pressure up to 280/160 mm Hg. The day before she experienced exhausting headache,
vascular pulsation, palpitations, asphyxia, stomachache, unbearable fear of coming death. The
patient paled and broke out in cold sweat. In urine there is increased content of vani-
llylmandelic acid. What disease causes such clinical presentation in the patient?
A. Pheochromocytoma*
B. Conn’s syndrome (primary hyperaldosteronism)
C. Cushing’s syndrome
D. Primary hypertension
E. Cushing’s disease

150. A 52-year-old patient suffers from marked dyspnea during physical exertion, non-productive
cough. The patient’s condition has been persisting for 8 months. The patient has been a
smoker for 30 years. In the lungs there are cellophane-type crackles auscultated on both sides.
Respiration rate is 26/min., oxygen saturation of blood is 92%. On spirometry: moderate
restrictive-type disturbance of external respiration. What is the most likely diagnosis?
A. Idiopathic fibrosing alveolitis*
B. Chronic obstructive pulmonary disease (COPD)
C. Chronic bronchitis
D. Community-acquired pneumonia
E. Sarcoidosis

151. A 26-year-old patient with affective bipolar disorder has developed a condition manifested by
mood improvement, behavioural and sexual hyperactivity, verbosity, active body language,
reduced need for sleep. Which of the following drugs would be most effective in this case?
A. Neuroleptics with sedative effect*
B. Antidepressants with activating effect
C. Neuroleptics with activating effect
D. Tranquilizers
E. Antidepressants with sedative effect

152. A 19-year-old patient complains of dyspnea on exertion. He often has bronchitis and
pneumonia. Since childhood the patient presents with cardiac murmur. Auscultation revealed
splitting of the II sound above the pulmonary artery, systolic murmur in the 3rd intercostal
space at the left sternal border. ECG detected right bundle branch block. What is the
provisional diagnosis?
A. Atrial septal defect*
B. Open ductus arteriosus
C. Aortarctia

35
Завантажено з сайту https://тестування.укр/ - онлайн тестування КРОК

D. Aortic stenosis
E. Mitral insufficiency

153. A 30-year-old patient was in a car accident. He is unconscious, pale, has thready pulse. In the
middle third of the right thigh there is an extensive laceration with ongoing profuse external
arterial bleeding. What urgent actions must be taken to save the life of the patient?
A. Tourniquet above the wound of the right thigh*
B. Tourniquet below the wound of the right thigh
C. Artificial lung ventilation
D. Precordial thump
E. Application of plaster bar

154. A 25-year-old patient has been admitted to the hospital with the following problems: weakness,
sweating, itching, weight loss, enlarged submandibular, cervical, axillary, inguinal lymph
nodes. Objectively: hepatomegaly. Lymph node biopsy revealed giant Berezovsky-Reed-
Sternberg cells, polymorphocellular granuloma composed of lymphocytes, reticular cells,
neutrophils, eosinophils, fibrous tissue, and plasma cells. What is the most likely diagnosis?
A. Lymphogranulomatosis (Hodgkin’s lymphoma)*
B. Lymph node tuberculosis
C. Lymphoreticulosarcoma
D. Cancer metastases to the lymph nodes
E. Macofollicular reticulosis

155. An 9-year-old child was hospitalized for fever up to 39,8oC, inertness, moderate headache,
vomiting. Examination revealed meningeal symptoms. Lumbar puncture was performed. The
obtained fluid was characterised by increased opening pressure, was transparent, with the cell
count of 450 cells per 1 mcL (mainly lymphocytes - 90%), glucose level of 3,6 mmol/l. What
agent could have caused the disease in the child?
A. Enterovirus*
B. Neisseria meningitidis
C. Mycobacterium tuberculosis
D. Staphylococcus aureus
E. Streptococcus pneumoniae

156. A 25-year-old woman has a self-detected tumor in the upper outer quadrant of her right breast.
On palpation there is a painless firm mobile lump up to 2 cm in diameter, peripheral lymph
nodes are without alterations. In the upper outer quadrant of the right breast ultrasound
revealed a massive neoplasm with increased echogenicity sized 21x18 mm. What is the most
likely diagnosis?
A. Fibroadenoma*
B. Lactocele
C. Diffuse mastopathy
D. Mammary cancer
E. Mastitis

157. A 68-year-old man complains of inability to urinate for a day. On attempt of urinary bladder

36
Завантажено з сайту https://тестування.укр/ - онлайн тестування КРОК

catheterization there was detected a rough stricture in the membranous portion of the
urethra. What first aid tactics should be applied in this case?
A. Troacar cystostomy*
B. Adenomectomy
C. Optical internal urethrotomy
D. a-adrenoblockers
E. Urinary bladder catheterization

158. Heart X-ray of a 31-year-old man has revealed the following: with tightly filled opacified
esophagus there is a marginal filling defect in its middle third on the posterior wall; the defect
is 1,8x1,3 cm in size with clear oval border. Mucosal folds are retained and envelop the defect;
wall peristalsis and elasticity are not affected. There are no complaints regarding the condition
of the patient’s alimentary canal. Make the provisional diagnosis:
A. Esophageal tumor*
B. Achalasia cardiae
C. Esophageal burns
D. Diverticulum
E. Barrett esophagus

159. A patient with signs of general overexposure to cold presenting with local frostbites of fingers
has been delivered into an admission room. Objectively: conscious, inert, speech is slow, the
skin of the face is cold, body temperature is 34oC, heart rate is 68/min. What would be the
actions of a doctor on call?
A. Hospitalize the patient to the surgical department*
B. Hospitalize the patient to the therapeutics department
C. Hospitalize the patient to the traumatology department
D. Let the patient go home
E. Refer to a family doctor on the next day

160. A 22-day-old infant developed subcutaneous red nodes from 1,0 to 1,5 cm in size on the scalp;
later the nodes suppurated. Temperature increased up to 37,7°C, intoxication symptoms
appeared, regional lymph nodes enlarged. Complete blood count: anemia, leukocytosis,
neutrocytosis, increased ESR. What diagnosis will you make?
A. Pseudofurunculosis*
B. Pemphigus
C. Vesiculopustulosis
D. Scalp phlegmon
E. -

161. During examination of a healthy infant, the child takes a toy into his hands, turns from the
back to the side; when lying on the stomach he can firmly prop himself up on his forearms; the
child laughs and makes joyful exclamations. The age of the child is:
A. 4 months*
B. 2 months
C. 5 months
D. 1 month
E. 6 months

37
Завантажено з сайту https://тестування.укр/ - онлайн тестування КРОК

162. After significant physical exertion a 66-year-old man with deep vein thrombosis of the
extremities developed shortness of breath, intense pain in the chest on the left, marked
palpitations. The patient’s condition is grave, his face is cyanotic, the cervical veins are
swollen, BP is 60/40 mm Hg. What investigation method would be the most advisable in this
case?
A. Selective angiopneumography*
B. Chest X-ray
C. Echocardiography
D. Magnetic resonance imaging of the chest
E. Fiber-optic bronchoscopy

163. A 32-year-old woman complains of general fatigue, low-grade fever persisting for 4 months,
lumbar pain, and dysuria. Anamnesis includes frequent acute respiratory diseases,
overexposure to cold, low-calorie diet, a case of pulmonary tuberculosis in childhood. Clinical
urine analysis: pH4,8, leukocyturia, hematuria. Complete blood count: leukocytosis,
lymphocytosis, increased ESR. Urography concludes: dilatation of renal pelvis and calyceal
system of both kidneys, foci of calcification in the projection of right kidney parenchyma. What
is the most likely diagnosis?
A. Nephrotuberculosis*
B. Right renal cyst
C. Right renal carcinoma
D. Acute glomerulonephritis
E. Chronic pyelonephritis

164. A full-term newborn (born with the body weight of 3900 g at gestational age of 39 weeks) on
the first day of his life developed respiratory disturbances: dyspnea, arrhythmic respiration,
cyanosis attacks. On examination there is paradoxical respiration observed and left side of the
chest lags behind in the act of breathing. On auscultation the respiration is weakened in the
lungs on the left. Neurologist diagnosed the patient with left-sided Erb-Duchenne palsy.
Complete blood count shows no changes. What is the most likely diagnosis?
A. Left-sided diaphragm paresis*
B. Congenital pneumonia
C. Left-sided pneumothorax
D. Respiratory distress syndrome
E. Transient tachypnea of the newborn

165. A 12-year-old child had three attacks of acute rheumatic fever accompanied by carditis.
Examination revealed the symptoms of chronic tonsillitis, mitral insufficiency, and carious
teeth. What is the optimal method of secondary prophylaxis?
A. Year-round bicillin prophylaxis until the age of 25*
B. Course of cardiotrophic drugs twice a year
C. Year-round bicillin prophylaxis for 3 years
D. Tonsillectomy
E. Oral cavity sanitation

166. A 24-year-old patient had been delivered to a thoracic department with a chest injury, fracture
of the IV, V VI ribs on the right. Plan radiography showed the fluid level in the pleural cavity to

38
Завантажено з сайту https://тестування.укр/ - онлайн тестування КРОК

be reaching the III rib on the right. Puncture contained blood clots. What is the optimal
treatment tactics?
A. Emergency thoracotomy*
B. Pleural puncture
C. Thoracentesis and thoracostomy
D. Hemostatic therapy
E. Medical thoracoscopy

167. A 3-year-old child has been delivered to a hospital with complaints of pain in the legs, fever,
loss of appetite. Objectively: pale skin and mucosa, hemorrhagic rash. Lymph nodes are
enlarged, painless, dense and elastic, not matted together. Bones, joints, and abdomen are
painful. The liver and spleen are enlarged. Hemogram: Hb- 88 g/l, color index - 1,3, platelets -
80 • 109/l, leukocytes - 25,8 • 109/l, lymphoblasts - 70%, ESR- 52 mm/hour. Make the
provisional diagnosis:
A. Acute leukemia*
B. Thrombocytopenic purpura
C. Acute rheumatic fever
D. Infectious mononucleosis
E. Hemorrhagic vasculitis (Henoch-Schonlein purpura)

168. A 60-year-old man has a diet consisting of unvaried food staples: mostly cereals, potato, pasta;
few vegetables and little fats (especially animal fats). During medical examination he
complains of deterioration of his twilight vision. This condition can be caused by lack of:
A. Retinol*
B. Amino acids
C. Fats
D. Calcium
E. Carbohydrates

169. A 14-year-old girl came to a general practitioner with complaints of weakness, loss of appetite,
headache, rapid fatigability. Her last menstruation was profuse and lasted for 14 days after
previous delay of 2 months. Objectively: the skin is pale, heart rate is 90/min., BP is 110/70
mm Hg, Hb is 88 g/l. Rectal examination: the uterus and its appendages are without changes,
no discharge from the genital tracts. What complication occurred in the patient?
A. Posthemorrhagic anemia*
B. Somatoform autonomic dysfunction of hypotonic type
C. Migraine
D. Gastritis
E. Dysmenorrhea

170. A postpartum woman on the 12th day after the normal delivery complains of pain localized in
her left gastrocnemius muscle. Body temperature is 37,2oC; pulse is 85/min, rhythmic; blood
pressure is 128/80 mm Hg. Mammary glands are soft and painless. The uterus is behind the
pubis. The left leg in the area of gastrocnemius muscle is by 3 cm larger than the right leg in
the diameter. Internal organs present no pathologies. What complication can be suspected?
A. Deep vein thrombosis of the shin*
B. Iliofemoral thrombosis

39
Завантажено з сайту https://тестування.укр/ - онлайн тестування КРОК

C. Varicose veins of lower extremities


D. Endometritis
E. Myositis

171. Examination of a Rh-negative pregnant woman at 32 weeks of gestation revealed a four-time


rise of Rh-antibody titer within 2 last weeks; the titer is 1:64. The first two pregnancies
resulted in antenatal fetal death due to hemolytic disease. What is the optimal tactics of
pregnancy management?
A. Early delivery*
B. Delivery at 37 weeks of gestation
C. Screening for Rh-antibodies in 2 weeks and urgent delivery in case of further increase of
antibody titer
D. Introduction of anti-Rh (D) immunoglobulin
E. Ultrasound for signs of hemolytic disease of the fetus

172. During last several weeks an 11-year-old girl has been complaining of dyspnea and edema of
shins and feet after physical exercise. After a long rest or sleep through the night her edemas
diminish significantly. On clinical examination there are enlarged liver and rasping systolic
murmur over the cardiac area. Blood and urine analyses are without changes. What is the
most likely cause of the child’s edema?
A. Heart failure*
B. Angioneurotic edema
C. Acute pyelonephritis
D. Hepatocirrhosis
E. Nephrotic syndrome

173. A 74-year-old man visited an urologist with complaints of pain above the pubis and inability to
urinate for 8 hours. At home he had taken antispasmodics and had a warm bath but no
improvement occurred. Objectively: the abdomen is soft and painful above the pubis; dullness
of percussion sound is observed above the pubis. Murphy’s punch sign (costovertebral angle
tenderness) is negative on both sides. What condition does the patient have?
A. Acute urinary retention*
B. Paradoxal ischuria
C. Chronic urinary retention
D. Anuria
E. Oliguria

174. During preventive examination a 58-year-old man on chest X-ray presents with multiple
globular pale shadows 3 cm in diameter within parenchyma of the both lungs. Examination in
the oncologic hospital: the primary focus is not found; transbronchial biopsy with cytologic
investigation detected cells of glandular neoplasm. What tactics should the physician choose?
A. Polychemotherapy courses*
B. Exploratory laparotomy
C. Exploratory thoracotomy
D. Laparoscopy
E. Symptomatic treatment at home

40
Завантажено з сайту https://тестування.укр/ - онлайн тестування КРОК

175. A 36-year-old man developed a disease with acute onset 6 hours ago. The patient presents with
pain in the epigastric, ileocecal, and paraumbilical areas, vomiting, weakness, nausea, and
body temperature of 38,5oC. Stool is liquid, profuse, frequent, retains fecal nature, foul-
smelling, frothy, colored dark green. The stomach is moderately distended and painful on
palpation. The patient attributes his disease to eating raw chicken eggs one day before the
clinical signs of the disease appeared. What is the most likely diagnosis?
A. Salmonellosis*
B. Shigellosis
C. Typhoid fever
D. Cholera
E. Food toxicoinfection

176. A 55-year-old woman came to a gynecologist with complaints of leukorrhea and bloody
discharge from the vagina after 5 years of menopause. Anamnesis states no pregnancies.
Bimanual examination: the uterus and uterine appendages are without changes. During
diagnostic curettage of the uterine cavity the physician scraped off enchephaloid matter. What
is the most likely diagnosis in this case?
A. Endometrial carcinoma*
B. Adenomyosis
C. Subserous uterine myoma
D. Cervical carcinoma
E. Ovarian carcinoma

177. A 10-year-old boy is delivered into a polytrauma unit after he received a blunt trauma of the
thorax, having fallen from the bicycle. Upon hospitalization his blood pressure is 110/80 mm
Hg, heart rate is 96/min. Chest X-ray is noncontributive to the diagnosis. Echocardiogram
shows free liquid in the pericardial cavity, in the amount of up to 100 ml. In an hour after the
hospitalization the patient started to develop increasing signs of heart failure: jugular venous
distention, decreased blood pressure down to 90/70 mm Hg, tachycardia up to 120/min. On
auscultation muffled heart sounds. What would be the primary tactics of a physician?
A. Pericardiocentesis*
B. Cardiac glycosides intravenously
C. Constant oxygenotherapy
D. Diuretics intravenously
E. Antibiotics intravenously

178. After the celebratory feast that took place the day before, a 35-year-old man was hospitalized
with complaints of marked pain within the I metatarsophalangeal articulation on the right,
which developed late in the night, and impaired walking. Objectively: the metatarsophalangeal
articulation is swollen, hyperemic, hot to touch, painful on movement. In blood: erythrocytes
-5,1 • 1012 /l, Hb-155 g/l, leukocytes-13,0 -109/l, ESR- 50 mm/hour, CRP- 46 mg/dl, uric acid -
720 mcmol/l. X-ray of feet articulations: osteoporosis, narrowing of interarticular spaces,
numerous punched-out erosions. Make the preliminary diagnosis:
A. Gout*
B. Osteoarthritis
C. Reactive arthritis
D. Rheumatoid arthritis
E. Psoriatic arthritis

41
Завантажено з сайту https://тестування.укр/ - онлайн тестування КРОК

179. A woman in her early- to mid-thirties has lost her consciousness 3-5 minutes ago. On
examination: the skin is pale, no pulse over the carotid arteries, no spontaneous respiration,
pupils are dilated; the patient is nonresponsive, presents with atony. The patient’s condition
can be determined as:
A. Apparent death*
B. Natural death
C. Syncope
D. Brain death
E. Coma

180. A 32-year-old woman complains of body weight loss despite her increased appetite,
nervousness, and tremor of the extremities. Objectively: the skin is moist; the thyroid gland is
diffusely enlarged, painless, soft, and mobile. Blood test: increased level of T3, T4, and thyroid-
stimulating hormone (THS). What is the most likely diagnosis?
A. Diffuse toxic goiter*
B. Thyroid carcinoma
C. Autoimmune (Hashimoto’s) thyroiditis
D. Thyroid adenoma
E. Diffuse nontoxic goiter

181. A 57-year-old patient complains of sensation of dryness and pain during swallowing, frequent
unbearable cough, the voice is hoarse. Disease onset was abrupt. On laryngoscopy: laryngeal
mucosa is hyperemic, vocal folds are swollen, laryngeal lumen contains viscous secretion.
What diagnosis is it?
A. Acute laryngitis*
B. Acute stenosing laryngotracheitis
C. Bronchial asthma
D. Flegmonous laryngitis
E. Laryngeal diphtheria

182. A 24-year-old pregnant woman on her 37th week of pregnancy has been delivered to a
maternity obstetric service with complaints of weak fetal movements. Fetal heartbeats are
95/min. On vaginal examination the uterine cervix is tilted backwards, 2 cm long, external
orifice allows inserting a fingertip. Biophysical profile of the fetus equals 4 points. What tactics
of pregnancy management should be chosen?
A. Urgent delivery via a cesarean section*
B. Treatment of placental dysfunction and repeated analysis of the fetal biophysical profile
on the next day
C. Doppler measurement of blood velocity in the umbilical artery
D. Urgent preparation of the uterine cervix for delivery
E. Treatment of fetal distress, if ineffective, then elective cesarean section on the next day

183. During regular preventive gynecological examination a 30-year-old woman was detected to
have dark blue punctulated ’’perforations” on the vaginal portion of the uterine cervix. The
doctor suspects endometriosis of the vaginal portion of the uterine cervix. What investigation
method would be most informative for diagnosis confirmation?
A. Colposcopy, target biopsy of the cervix*

42
Завантажено з сайту https://тестування.укр/ - онлайн тестування КРОК

B. US of small pelvis
C. Hysteroscopy
D. Curettage of the uterine cavity
E. Hormone testing

184. A 26-year-old woman came to a gynecologist for a regular check-up. She has no complaints.
Per vaginum: the uterus lies in anteflexion, not enlarged, dense, mobile, painless. On the left
from the uterus in the area of uterine appendages there is a mobile painless outgrowth that
can be moved independently from the uterus. On the right the appendages cannot be detected.
What additional investigation would be informative for diagnosis clarification?
A. US of lesser pelvis*
B. Metrosalpingography
C. Examination for urogenital infection
D. Colposcopy
E. Colonoscopy

185. A 57-year-old patient complains of dyspnea at rest. The patient presents with orthopnea,
acrocyanosis, bulging cervical veins. On percussion: dull sound over the lower lung segments;
on auscultation: no respiratory murmurs. Heart rate is 92/min. Right-sided cardiac dilatation is
observed. The liver is enlarged by 7 cm. Shins are swollen. Pleural effusion is suspected. What
indicator would confirm the presence of transudate in this case?
A. Total protein content in the pleural fluid below 25 g/l*
B. Presence of atypical cells
C. Total protein content in the pleural fluid above 30 g/l
D. Specific gravity exceeding 1015
E. Positive Rivalta’s test

186. A 28-year-old woman complains of girdle pain in her epigastric and left subcostal areas with
irradiation to the back, nausea, and vomiting without relief. On examination a surgeon
observes stomach distension and meteorism. There are positive Mondor’s, Mayo-Robson’s, and
Cullen’s symptoms. What is the most likely diagnosis?
A. Acute pancreatitis*
B. Acute cholecystitis
C. Acute intestinal obstruction
D. Aortic dissecting aneurysm
E. Splenic infarction

187. A 36-year-old woman complains of pain in her lumbar area, which irradiates to her lower right
limb and increases during movements, and sensation of numbness in her limb. Objectively:
palpation of the shin and thigh muscles is painful, positive stretch symptom on the right. MRI
scan: herniation of intervertebral disk L5-S1 4 mm in size. What is the most likely diagnosis?
A. Vertebrogenic lumbago*
B. Vertebrogenic radicular syndrome of L5-S1 on the right
C. Endarteritis of lower extremities
D. Spinal stroke
E. Acute myelitis

43
Завантажено з сайту https://тестування.укр/ - онлайн тестування КРОК

188. During routine medical examination a 35-year-old woman presents with enlarged cervical and
mediastinal lymph nodes. Her overall health is satisfactory. ESR is 30 mm/hour. Cervical node
biopsy was performed. In the specimen there are granulomas composed of epithelial and giant
cells, no caseous necrosis detected. What is the most likely diagnosis?
A. Sarcoidosis*
B. Lymphogranulomatosis
C. Infectious mononucleosis
D. Nonspecific lymphadenitis
E. Lymph node tuberculosis

189. A 39-year-old woman complains of violent pain in her left lumbar area with irradiation to the
right iliac area. Several years ago she was diagnosed with cholelithiasis and urolithiasis. The
patient’s condition is moderately severe, the skin is dry. Ortner’s symptom is negative;
costovertebral angle tenderness is observed on the right. The most reasonable treatment
tactics would be:
A. Spasmolytics and analgesics*
B. Urgent hemodialysis
C. Peritoneal dialysis
D. Laparoscopic cholecystectomy
E. Antibiotics

190. An 18-year-old patient complains of skin rash. The patient has been suffering from this
condition for 5 years. The first instance of this disease occurred after a car accident.
Objectively: the patient presents with papular rash covered in silvery scales, ”thimble”
symptom (small pits on the nails), affected joints. What is the most likely diagnosis?
A. Psoriasis*
B. Panaritium
C. Onychomycosis
D. Lupus erythematosus
E. Rheumatism

191. A woman complains of frequent, liquid stool (up to 9-10 times per day) with mucus and blood
admixtures, dull pain in the hypogastrium, weight loss of 4 kg within the last year. Objectively:
malnutrition, dry skin, low turgor, aphthous stomatitis. The stomach is soft, the sigmoid colon
is spastic and painful on palpation. Occult blood test is positive. Fi-brocolonoscopy: edema,
hyperemia, mucosal granulation, pseudopolyps, small ulcers with irregular edges. Make the
diagnosis:
A. Nonspecific ulcerative colitis*
B. Chronic enterocolitis
C. Colon cancer
D. Irritable bowel syndrome
E. Crohn’s disease (regional enteritis)

192. A 48-year-old woman has been hospitalized due to development of tachysystolic atrial
fibrillation. She has lost 5 kg of body weight within 2 months. On palpation there is a node in
the left lobe of the thyroid gland. What pathology resulted in the development of this
condition?

44
Завантажено з сайту https://тестування.укр/ - онлайн тестування КРОК

A. Toxic nodular goiter*


B. Aterosclerotic cardiosclerosis
C. Chronic thyroiditis
D. Nontoxic nodular goiter
E. Autoimmune thyroiditis

193. On the 9th day after childbirth the obstetric patient developed high fever up to 38oC. She
complains of pain in the right mammary gland. On examination a sharply painful infiltrate can
be palpated in the right mammary gland, the skin over the infiltrate is red, subareolar area
and nipple are swollen and painful. What is your diagnosis?
A. Abscess of the right mammary gland*
B. Mastopathy
C. Cancer of the right mammary gland
D. Serous mastitis
E. Fibrous cystic degeneration of the right mammary gland

194. A 32-year-old woman complains of episodes of intense fear that occur without visible cause
and last for 10-20 minutes; the episodes are characterized by rapid pulse, sweating, labored
breathing, and vertigo. Specify the likely diagnosis:
A. Panic disorder*
B. Paranoid syndrome
C. Manic syndrome
D. Simple schizophrenia
E. Claustrophobia

195. A resuscitation unit received a 46-year-old woman, who has been suffering from diabetes
mellitus type 1 for approximately 30 years. Objectively: the skin is pale, heart sounds are
weakened, BP is 170/100 mm Hg, lower limbs are markedly swollen. Blood creatinine -1125
mcmol/l, urea - 49,6 mmol/l, potassium - 6.3 mmol/l, glucose - 7,6 mmol/l, glomerular filtration
rate - 5 ml/min. What treatment is indicated for the patient in the first place?
A. Hemodialysis*
B. Kidney transplantation
C. Hemofiltration
D. Enterosorption
E. Conservative detoxification therapy

196. A woman is on the 32nd week of her second pregnancy. She complains of fever, chills, nausea,
vomiting, lumbar pain, and dysuria. Costovertebral angle tenderness is present on both sides.
Urine analysis: pyuria, bacteriuria. Blood test: leukocytosis. What is the most likely diagnosis?
A. Gestational pyelonephritis*
B. Cystitis
C. Pyelitis
D. Glomerulonephritis
E. Latent bacteriuria

197. A 19-year-old young man complains of cough with expectoration of purulent sputum in amount

45
Завантажено з сайту https://тестування.укр/ - онлайн тестування КРОК

of 100 ml per day, hemoptysis, dyspnea, increased body temperature up to 37,8oC, general
weakness, weight loss. The patient’s condition lasts for 4 years. Exacerbations occur 2-3 times
a year. The patient presents with malnutrition, pale skin, cyanosis of the lips, drumstick
(clubbed) fingers. Tympanic percussion sound in the lungs, weakened respiration, various
numerous moist crackles in the lower pulmonary segments on the left can be observed. In
blood: erythrocytes - 3,2 • 1012/l, leukocytes - 8,4 • 109/l, ESR- 56 mm/hour. On X-ray: lung
fields are emphysematous, the left pulmonary root is deformed and dilated. What is the most
likely diagnosis?
A. Multiple bronchiectasis of the left lung*
B. Chronic left-sided pneumonia
C. Chronic abscess of the left lung
D. Left-sided pulmonary cystic dysplasia
E. Suppuration of the cyst in the left lung

198. A 37-year-old man working as a typesetter in a print shop complains of rapid fatigability,
paroxysmal attacks of stomachache, weak drooping hands. Examination of neurological status
revealed hypotrophy of the forearm muscles. Carporadial reflexes are sharply weakened.
Sensitivity is not disturbed. Gums present with dark blue border. What neurological pathology
is it?
A. Lead polyneuropathy*
B. Guillain-Barre syndrome (postinfectious polyneuritis)
C. Shingles
D. Ulnar neuropathy
E. Brachial plexitis

199. A 54-year-old woman takes antihypertensive drugs for hypertension. Having discovered that
her son was arrested and is under investigation, became agitated and extremely anxious. She
lost her orientation in place, stopped recognizing her relatives, started hearing ’’voices”
threatening her and her son with violence. She had opened her window (on the 8th floor) and
tried to jump out, resisted the people, who were holding her back. What drugs should be
administered to terminate such condition of the patient?
A. Antipsychotics*
B. Antidepressants
C. Tranquilizers
D. Nootropic agents
E. Antihypertensive drugs

200. A 43-year-old man, who has been abusing alcohol and suffering from pulmonary tuberculosis,
in the course of two weeks gradually developed general weakness, headache, diplopia,
vomiting. Objectively: ptosis on the left, anisocoria S>D, exotropia of the left eye, neck
stiffness; Kernig’s and Brudzinski’s signs are positive. In cerebrospinal fluid: lymphocytic
pleocytosis, low glucose, precipitation of cerebrospinal fluid resulted in production of fibrin
film. What is the most likely diagnosis?
A. Tuberculous meningitis*
B. Subarachnoid hemorrhage
C. Brainstem encephalitis
D. Acute myelitis
E. Basal arachnoiditis

46
Завантажено з сайту https://тестування.укр/ - онлайн тестування КРОК

47

You might also like